You are on page 1of 476

Examen de Pediatría

ENAM 2014 capítulo 5


Juan José García Bustinza
Nefrólogo Pediatra
HNERM-UPCH
Preguntas de Pediatría
1. Un joven de 13 años consultó por la aparición, 8 días
antes, de una placa oval en región dorsal del tórax, de 3 cm de
diámetro aproximado, de tono rojo anaranjado, con borde
levemente descamativo. El día previo a la consulta
aparecieron, también en tórax, otras placas de similar
configuración, pero más pequeñas y pruriginosas. ¿Cuál es el
diagnóstico MÁS probable del paciente?
A. Nódulos escabióticos
B. Pitiriasis rosada de Gibert
C. Psoriasis
D. Megaloeritema
E. Pitiriasis alba
Pitiriasis rosada
• Es una enfermedad de la piel autolimitada. Agentes virales como el
Herpes virus tipo 6 y 7 pueden considerarse como posibles causas,
aunque su etiología permanece desconocida.
• Afecta típicamente a niños y adultos jóvenes. Se caracteriza por
presentar una placa heráldica inicial, seguida de máculas color
asalmonadas con escamas.
• Varias medicaciones pueden provocar una erupción símil
ocasionando un cuadro más extendido y duradero.
• La distribución típica es el tronco, abdomen, y parte proximal de
extremidades. No obstante las lesiones pueden aparecen en
cualquier parte del cuerpo hasta en las palmas.
• El prurito es relativamente común, y es severo en el 25 % de los
pacientes
Pitiriasis rosada
Tratamiento de la Pitiriasis rosada
• Sintomático, se pueden utilizar baños suaves,
lubricantes o cremas suaves, o cremas con
hidrocortisona suaves
• Para reducir la picazón, se pueden utilizar
antihistamínicos orales
• La exposición moderada a los rayos de sol o el
tratamiento con luz ultravioleta pueden ayudar a hacer
que las lesiones desaparezcan con mayor rapidez
• La pitiriasis rosada desaparece normalmente en
cuestión de 6 a 12 semanas y, por lo general, no
reaparece
CLAVE B
Preguntas de Pediatría
2. Lo consultan por un paciente de 10 años que presenta una
tiña capitis con 48 horas de tratamiento con griseofulvina.
¿Cuál es la conducta a adoptar respecto a la asistencia al
colegio?:
A. No debe concurrir hasta el fin del tratamiento
B. No debe concurrir hasta 15 días después del comienzo del
tratamiento
C. Puede concurrir cuando crezcan los cabellos normales
d) Puede concurrir desde el momento de la consulta
e) Puede concurrir desde las 48 horas de iniciado el
tratamiento
TIÑA CAPITIS

• Mayor frecuencia en la infancia por zoófilos


como el M. canis
• Placa alopécica de aspecto sucio con escamas
grisáceas
• Clínicamente hay 3 tipos:
Tonsurantes (trichophyton o microsporum)
Supuradas (Querion)
Fávica
TIÑA TONSURANTE
Microspórica Tricofítica
90% 10%
• Pocas placas grandes • Más placas pero
(4,5 cm) pequeñas
• Alternan pelos sanos y
• Todos los pelos a 3-4mm enfermos cortados a
cortados al mismo nivel. distintos niveles
• Pelo ralo se desprende • Pelo enroscado en una
fácilmente micropápula, “punto
• Esporos fuera del tallo negro”
piloso: ectotrix ,Wood + • Esporos dentro del tallo
piloso: endotrix ,Wood -
TIÑA CAPITIS

• Tiña supurada ó QUERION de CELSO:


Agentes zoófilos generalmente.
Placa única sobreelvada que a la compresión emana
pus por los orificios foliculares “signo de la
espumadera”.
Evolución subaguda.
Adenopatías
Tiña Capitis
Tiña tonsurante Querion de Celso
Tratamiento de la Tiña capitis
• La griseofulvina 20 a 25 mg/kg durante 2 a 3 meses es
el fármaco de elección.
• Inhibe la síntesis de ácidos nucleicos y tiene efecto
fungistático y lipolítico y su absorción es mejorada por
la ingestión de ácidos grasos
• Terbinafina, itraconazol y fluconazol tienen
porcentajes de eficacia similares a la griseofulvina en
niños con TC causados por Tricofitum, requiriendo una
duración de tratamiento más corta
• La griseofulvina es el tratamiento de elección para
Microsporum. Su eficacia es superior a terbinafina
Tratamiento de la Tiña capitis
• Lavado diario del cuero cabelludo con un champú
de sulfuro de selenio u otro champú antifúngico
para reducir la contagiosidad
• El niño no debe ir a la escuela hasta transcurridos
15 días
• El origen suele ser antropofílico por lo que debe
investigarse la existencia de animales infectados
(perros, gatos o conejos).
• Los antifúngicos tópicos no penetran el tallo del
pelo, y por tanto no son efectivos.
CLAVE B
Preguntas de Pediatría
3. La urticaria se caracteriza por presentar:
A. Vesículas
B. Habones
C. Pápulas eritematosas
D. Eccema
E. Pústulas
CARACTERISTICAS
URTICARIA:
►Erupción cutánea definida por la presencia del
habón.
►Pápula eritematosa, pruriginosa y evanescente
de menos de 24h de duración, en la misma
localización y desaparece sin dejar lesión
residual. Cursa en brotes.
►Desaparecen a la digitopresión.
►Se suele asociar al angioedema.
►Pueden distribuirse por
cualquier zona del cuerpo.
FISIOPATOLOGIA
URTICARIA/ANGIOEDEMA:

La liberación de mediadores mastocitarios por


distintos estímulos induce cambios vasculares con
vasodilatación, aumento de permeabilidad de
capilares y vénulas, con extravasación de fluido
y un reflejo nervioso que condiciona la aparición
de edema y eritema.
o Si es superficial = habones.
o Si es profunda = angioedema.
CLAVE B
Preguntas de Pediatría
4. Paciente de 10 años que después de haber
nadado presenta habones e hipotensión brusca que
mejora con abrigo. El diagnóstico es urticaria
causada por:
A. Agua
B. Sol
C. Frio
D. Droga
E. Idiopática
URTICARIA AGUDA
►Lesiones habonosas de una duración de menos
de 6 semanas.
►Persisten horas o días y desaparecen sin dejar
rastro.
►Es la presentación más frecuente (13-17% de
todas las urticarias).
►Las causas más frecuentes:
o Fármacos (AINES, Antibióticos betalactámicos…)
o Alimentos (Frutos secos, leche, huevos, crustáceos)
o Picaduras de himenópteros
o Otros (exposición masiva o de contacto a alergenos)
URTICARIA POR CALOR:
o Se presenta por contacto directo de la piel
con objetos o aire caliente.
o Los rangos de T° desencadenante varían de
38°C a 50°C.

URTICARIA POR FRIO:


o Inducida por exposición a bajas T° (aire,
agua, objetos, alimentos o bebidas).
o Se cree que como mecanismo están
involucrados los Ac.
URTICARIA SOLAR:
o A los pocos minutos de la exposición a la luz
solar.
o Inducen urticaria las que van de los 2800 a
5000nm.
o Se basa en la activación de una molécula
precursora, que a la exposición se convierte
en foto-alérgeno.
URTICARIA ACUOGENICA:
o Tras el contacto con el agua.
o Lesiones urticarianas pequeñas.
o Se puede demostrar aplicando una toalla
húmeda durante 30 min.
CLAVE C
Preguntas de Pediatría
5. ¿Cuál de las siguientes condiciones
corresponde a reacción alérgica tipo IV?
A. Dermatitis de contacto
B. Enfermedad del suero
C. Urticaria
D. Síndrome de Stevens Jonhson
E. Hemólisis por isoanticuerpos
Tipos de hipersensibilidad
Tipo Característica Patologías frecuentes Mediadores
I Inmediata (alergia) Atopia Ig E
Anafilaxia
Asma
II Anticuerpo Anemia hemolítica Ig M o Ig G
dependiente autoinmune Complemento
Trombocitopenia
Eritroblastosis fetal
Síndrome de Goodpasture
Miastenia Gravis
III Enfermedad de Enfermedad del suero Ig G
complejos inmunes Reacción de Arthus Complemento
Lupus eritematoso sistémico
IV Citotóxica Dermatitis de contacto Linfocito T
Hipersensibilidad PPD
retardada Prueba de Montenegro
Rechazo crónico
Esclerosis múltiple
CLAVE A
Preguntas de Pediatría
6. En el eccema atópico se presenta:
A. Erupción pruriginosa vesicular liquenificada
B. Lesiones eritematosas cubiertas de escamas
C. Eritema vesicular en palma de manos
D. Vesículas pequeñas de límites netos
E. Lesiones papulosas en zonas de contacto
Dermatitis atopica

Es una enfermedad
inflamatoria de la piel en
la que se destaca:
eritema, prurito intenso,
exudación, formación de
costras y descamación
de las mismas.
Liquenificación:
Engrosamiento de la piel
con desarrollo de una
superficie seca y áspera,
como consecuencia de
un rascado repetido
CARACTERIZACION SEGÚN LESIONES

• La DA aguda se caracteriza por intenso


prurito, pápulas eritematosas asociadas
con excoriación, vesículas y exudado
seroso.
• La DA subaguda por pápulas
eritematosas, escamosas y excoriadas.
• La DA crónica por placas engrosadas,
acentuación del relieve cutáneo y pápulas
fibróticas.
CLAVE A
Preguntas de Pediatría
7. En relación a dermatitis atópica: es un criterio
mayor.
a. Palma hiperlinear
b. Línea de Dennie’s Morgan.
c. Prurito.
d. Xerosis.
e. Incremento serico de IgE.
Se precisan tres o más criterios mayores y tres o más criterios menores. Hay que tener en cuenta que en los niños hasta
los 2 años muchos de los criterios menores no se pueden aplicar.
CLAVE C
Preguntas de Pediatría
8. El eczema atópico en los lactantes tiene
una distribución preferente en:
A. Muñecas
B. Pliegues cutáneos
C. Cuello
D. Tobillos
E. Cuero cabelludo
Distribución según grupo etáreo
Dermatitis atópica del lactante
CLAVE E
Preguntas de Pediatría
9. La formación de escamas y costras difusas
en el cuero cabelludos “costra láctea” constituye
la manifestación inicial de:
A. Eczema numular
B. Pitiriasis alba
C. Liquen simple crónico
D. Eczema dishidrótico
E. Dermatitis seborreica
Dermatitis seborreica
• Es una afección inofensiva y temporal.
• Aparece en el cuero cabelludo del niño en
forma de escamas cafés o amarillas, gruesas y
costrosas. También se pueden encontrar
escamas similares en los párpados, los oídos,
alrededor de la nariz y en la ingle.
• La costra láctea se puede observar en recién
nacidos y niños pequeños hasta los 3 años.
Dermatitis seborreica
CLAVE E
Preguntas de Pediatría
10. Usted recibe a un lactante de 8 meses de edad con placas eritemato-
costrosas en sus mejillas. El resto del examen físico sólo revela una piel seca
generalizada. Estas manifestaciones han tenido un curso crónico, con
exacerbaciones y remisiones. El interrogatorio revela una historia personal de
episodios de broncoespasmos recurrentes y una historia familiar de atopía. La
niña no parece estar molesta. ¿Cuál es el tratamiento que indicaría como
primera línea?
A. Aplicación de cremas emolientes e hidrocortisona local al 1-2 % 2
veces al día durante una a dos semanas continuas
B. Aplicación de cremas emolientes y mometasona local una vez al día
durante cuatro semanas continuas
C. Aplicación de cremas emolientes y metilprednisona 1 mg/k/d via
oral por cuatro días consecutivos
D. Aplicación de cremas emolientes y fluticasona local 3 veces al día
durante cuatro semanas continuas
E. Prednisona via oral por 8 semanas
Tratamiento de la dermatitis atópica
• El tratamiento se realiza con cremas hidratantes y
productos emolientes ( que “ablandan” la capa
córnea). Se utilizarán una o dos veces al día, siendo
conveniente que una de las dos sea tras el baño
• En pieles muy sensibles o si existe brote agudo
especialmente con eczema exudativo, las sustancias
emolientes, como la urea y algunos ácidos resultan
irritantes y molestos para el niño.
• En zonas inflamadas es preciso utilizar los
corticosteroides tópicos (cuyos excipientes pueden
colaborar a la hidratación de la piel) y aplicar los
emolientes cuando la inflamación esté controlada
CLAVE A
Preguntas de Pediatría
11. Niño de 3 años con aparición aguda de prurito
intenso, al examen pápulas con un área eritematosa algo
elevada alrededor en los miembros inferiores y en el
tronco, algunas con una pequeña vesícula central y otras
con escoriaciones. ¿Cuál es el diagnóstico más probable?
A. Impetigo
B. Acarosis
C. Prurigo
D. Dermatitis atópica
E. Varicela
PRURIGO
• Síndrome reaccional cutáneo caracterizado por
pápulas y signos de prurito: Costras hemáticas y
liquenificacion.

Rascado

Múltiples Topografía
depende de Prurito
causas la causa Intenso

Lesión
Impétigo
TOPOGRAFÍA
• Se puede presentar en todo el cuerpo, pero su
distribución depende del agente.

Tronco, ambas Cara y áreas


caras, desprovistas de
sobretodo en ropa.
áreas
cubiertas.
• Chinches, las lesiones son
lineales y en pares.
• Pulgas, dispersas.
PATOGENESIS
• Las lesiones no aparecen por el piquete, sino por una acción de
sensibilización a estas sustancias.

• Respuesta de hipersensitividad precoz (I) y tardía (IV).

 Precoz (I):
• IgE e histamina  edema vasomotor transitorio  roncha

• Tardía (IV):
• Linfocitos T  infiltrado linfohistiocitico  pápulas.

*No todas las lesiones son por efecto directo de la picadura. Sino, por fenómeno de
sensibilización que origina lesiones a distancia.
PATOGENESIS
• Las ronchas se
presentan al piquete del
insecto y desaparecen
pronto.
 Las pápulas, tardan mas en
aparecer y desaparecer, (2-
3mm)
 Muchas veces decapitadas
por el rascado.
 Ampollas hasta de 1cm en
Reacción de hipersensibilidad.
CLAVE C
Preguntas de Pediatría
12. En el tratamiento del prurigo agudo usted no
recomendaría
A. Fumigar el dormitorio del niño
B. Evitar el contacto con los animales
C. Baño diario obligatorio del niño antes de
acostarlo
D. Corticoides sistémicos
E. Uso de antihistamínicos
Tratamiento del Prurigo
• Fumigar ambientes donde el niño
permanece
• Evitar el contacto con los
animales.
• Baño diario obligatorio del niño
antes de acostarlo y cambio de
ropas de dormir
• Antihistamínicos sistémicos:
loratadina, cetirizina, hidroxicina
• Corticoides tópicos de baja
potencia: hidrocortisona al 1 %.
• Antibióticos sistémicos en caso de
existir lesiones sobreinfectadas,
se puede usar crema de
mupirocina si el área no es
extensa
CLAVE D
Preguntas de Pediatría
13. Varón de 16 años, que presenta lesiones
papulo-pustulosas y quistes nodulares en tórax,
parte superior de la espalda y región deltoidea.
¿Cuál es el diagnostico?
A. Forunculosis
B. Dermatitis por esteroides
C. Impétigo
D. Herpes simple
E. Acné vulgar
Acné Adolescente (≥12 - ≤19 y/o después de la
menarquia en niñas)

• Epidemiología
– 85% de la población joven
– Varones más frecuente
– Baja incidencia en Asiáticos y Africanos
– Predisposición familiar en acné nóduloquístico
– Empeora en otoño e invierno
• Factores predisponentes
– Acne mecánica (manipulación), influyen hábitos
alimentarios?, estrés emocional.

Fitzpatrick´s. Color Atlas and Sinopsis of Clinical Dermatology. Sixth Edition.


Manifestaciones clínicas acné vulgar-Definición

• Severidad
• Comedoniano – Leve
• Pápulopustuloso – Moderado
• Nóduloquístico – Grave
• Variantes

Perioral Dermatitis. Chapter 4.4. Textbook of Pediatric Dermatology. Second Edition.


Clasificación de Severidad
• Acné leve:< 20 comedones o <15 pápulas
inflamatorias o un conteo de lesiones < 30
• Acné moderado: 15 a 50 pápulas y pústulas
con comedones y quistes. El rango total del
conteo de lesiones es de 30 –125,
• Acné severo: quistes y nódulos inflamatorios,
además de presencia de pápulas y pústulas
con un conteo total de lesiones mayor a 125.
Comedoniano Pápulopustuloso Nóduloquístico

Fitzpatrick´s. Color Atlas and Sinopsis of Clinical Dermatology. Sixth Edition.


CLAVE E
Preguntas de Pediatría
14. ¿Qué lesión caracteriza al acné juvenil grado
III?
A. Pústula
B. Pápulas
C. Comedones
D. Nódulos
E. Nódulo quiste
División del Acné juvenil
No inflamatorio Inflamatorio
• Grado I: comedoniano • Grado III: comedones,
pápulas y pústulas
• Grado II: comedones y
escasas pápulas • Grado IV (severo): se
agregan nódulos, quistes,
flemones y cicatrices
CLAVE A
Preguntas de Pediatría
15. En el tratamiento tópico de acné
comedosiano se usa:
A. Corticoides
B. Fototerapia
C. Retinol
D. Ácido salicílico
E. Calciprotol
Algoritmo de la Global Alliance para el tratamiento del Acne

Leve Moderado Grave

Comedoniano Papular/pustu Papular/pustu Nodular Nodular/Congl


loso loso obata
1ª Elección Retinoide tópico Retinoide Antibiótico Antibiótico Isotretinoína oral
tópico+Antimicrobian oral+retinoide tópico oral+retinoide tópico
o tópico +/- POB +POB
Alternativa
Retinoide tópico Antimicromiano Antimicromiano Isotretinoína oral ó Antibiótico oral a altas
alternativo tópico tópico Antimicromiano dosis+retinoide
ó ácido azelaico alternativo+retinoide alternativo+retinoide tópico tópico+POB
ó ácido salicílico tópico alternativo ó tópico alternativo ó alternativo+retinoide
ácido azelaico +/- POB tópico alternativo ó
Alternativa +/- POB/ácido azelaico
mujeres
Ver 1ªelección Ver 1ª elección Antiandrógeno Antiandrógeno Antiandrógeno oral a
oral+retinoide oral+retinoide tópico altas dosis +retinoide
tópico/ácido azelaico +/-antibiótico oral +/- tópico +/-
+/- antimicrobiano antimicrobiano antimicrobiano
Mantenimiento tópico alternativo alternativo
Retinoide tópico Retinoide tópico+/-POB

Actas Dermosifiliogr.2011;102(2):121-131.
CLAVE C
Preguntas de Pediatría
16. Niño de 4 años que presenta tos y dificultad
respiratoria todas las noches y con la actividad
física. ¿Cuál es el tratamiento a seguir?
A. Corticoide inhalado a altas dosis y beta 2 de
acción prolongada
B. Corticoide inhalado a baja dosis y adrenalina
C. Corticoide inhalado a baja dosis y cromonas
D. Bromuro de ipratropio y fluticasona
E. Corticoide inhalado de acción corta y teofilina
Niveles de control de asma
CLAVE A
Preguntas de Pediatría
17. En el tratamiento del asma bronquial, ¿Cuál
de los siguientes fármacos bloquea receptores
de leucotrienos?
A. Cromoglicato
B. Teofilina
C. Fluticasona
D. Formoterol
E. Montelukast
Antagonistas del receptor de
Leucotrienos CysLT1
CLAVE E
Preguntas de Pediatría
18. En un paciente de 8 años, que acude a
Emergencia con crisis asmática, ¿Cuál de los
siguientes signos indica que es severa?
A. Aleteo nasal
B. Taquipnea
C. Tirajes intercostales
D. Cianosis
E. Sibilancias en dos tiempos
EVALUACION DE SEVERIDAD
PARO
LEVE MODERADA SEVERA RESPIRATORIO
INMINENTE
DISNEA Caminando Hablando Reposo

HABLA EN Párrafos Frases Palabras

ESTADO Puede estar Usualmente Usualmente Somnoliento o


ALERTA agitado agitado agitado confuso

FR Incrementada Incrementada > 30/min

MÚSCULOS No usualmente Usualmente Usualmente Movimiento


ACCESORIOS Y paradójico
RETRACCIÓN
toraco-abdominal
SUPRAESTERNAL
EVALUACION DE SEVERIDAD
PARO
LEVE MODERADA SEVERA RESPIRATORIO
INMINENTE
SIBILANCIAS Moderada, final Intensas Usualmente Ausencia
espiración intensas

PULSO/MIN. < 100 100-120 >120 Bradicardia

PULSO Ausente Puede estar Frecuente Ausencia sugiere


PARADOJICO < 10 mm Hg 10-25 mm Hg > 25 mm Hg fatiga músculos
respiratorios
PEF Sobre 80% 60-80% < 60%(<100L/min)
o respuesta < 2h
PaO2 Normal > 60 mm Hg < 60 mm Hg
Posible cianosis
y/o PaCO2
< 45 mm Hg < 45 mm Hg > 45 mm Hg;
Posible falla resp.
SO2% > 95% 91-95% < 90%
CLAVE D
Preguntas de Pediatría
19. Escolar de 7 años de edad, asmático, acude a
emergencia por presentar tos y sibilancias. Al
evaluarlo se obtiene una puntuación de 6 en el
score de Bierman - Pearson. Selecciones el manejo
inicial:
a. Dexametasona y ambroxol
b. Salbutamol y aminofilina
c. Adrenalina y cromoglicato
d. Salbutamol y dexametasona
e. Aminofilina y cromoglicato
MANEJO EXACERBACION ASMA
EVALUACION INICIAL
 Historia, examen físico (auscultación, uso de músculos accesorios,
FC, FR, PEF o FEV1, saturación oxígeno, gases en sangre arterial)

TRATAMIENTO INICIAL
Oxígeno para lograr SO2 > 90
B2-agonistas acción rápida inhalados continuo durante 1 hora
Glucocorticoides sistémicos sin no hay respuesta inmediata, o si el paciente
ha tomado recientemente glucocorticoides orales, o si el episodio es severo
Sedación está contraindicada

Reevaluación después 1 hora


Examen físico, PEF, SO2 y otros test

Criterios para episodio moderado: Criterios para episodio severo:


PEF 60-80% Historia de factores de riesgo asma fatal
Examen físico: síntomas moderados, PEF < 60%
uso de músculos accesorios Examen físico: síntomas severos,
Tratamiento: retracción torácica
Oxígeno Ninguna mejoría luego tratamiento inicial
B2-agonistas inhalados y anticolinérgicos Tratamiento:
inhalados cada 60 min Oxígeno
Continue tratamiento por 1-3 horas, B2-agonistas inhalados y anticolinérgicos
inhalados
Glucocorticoides sistémicos
Magnesio
Re-evaluar después 1-2 horas

Buena respuesta dentro Respuesta incompleta dentro Pobre respuesta dentro 1-2h:
1-2 h: 1-2h:  Factores asma fatal
 Rpta. sostenida 60 min  Factores asma fatal  Síntomas severos,
 No distrés  Signos leves-mod. somnolencia, confusión
 PEF>60%  PEF <60%  PEV <30%
 O2 sat. >90%  O2 sat. sin mejoría  PCO2 > 45 mm Hg
_________________________  P02 < 60 mm Hg
Monitoreo estricto: __________________________
 Oxígeno UCI:
 b2-agonistas + anticolinérgicos  Oxígeno
 Glucocorticoides Sistémicos  B2-agonistas + anticolinérgicos
 Magnesio  Glucocorticoides Sistémicos
 Monitoreo PEF, Sat. O2, pulso  Considerar B2-agonistas IV
 Considerar teofilina IV
 Posible intubacion y VM

Reevaluación por intervalos


Criterios alta:
• PEF > 60% predicho/personal
• Medicación oral/inhalada Pobre respuesta
__________________________________ • Admitir Cuidados Intensivos
Tratamiento casa:
• B2-agonistas Respuesta incompleta 6-12h
• Glucocorticoides oral • Considerar admisión a
• Inhalación combinada cuidados intensivos
• Educación: Toma adecuada medicación
Revisar plan acción
Seguimiento médico estricto Mejora
CLAVE D
Preguntas de Pediatría
20. Lactante de 3 meses de edad previamente sano, inicia
su enfermedad con coriza y fiebre. Se agrega tos y 2 días
después es traído a emergencia por presentar dificultad
respiratoria. Al examen se auscultan roncantes y
subcrepitantes. ¿Cuál es el diagnóstico más probable?
A. Neumonía
B. Laringotraqueitis
C. Rinofaringitis
D. SOBA
E. Bronquiolitis
Bronquiolitis
• Enfermedad aguda de
etiología viral, que afecta
el aparato respiratorio en
forma difusa y bilateral, y
que se manifiesta clínica y
funcionalmente por
Incapacidad Ventilatoria
Obstructiva.
• Agentes etiológicos: VSR,
Adenovirus, Influenza
(tipo A y B), Parainfluenza
(tipo l, ll, lll) y Rhinovirus.
Fisiopatología de la Bronquiolitis
• Período de incubación 5 días
• Replicación en la nasofaringe y
luego en el tracto respiratorio
inferior
• Epitelio ciliado es el más
afectado
• En casos severos necrosis
extensa del epitelio
bronquiolar, infiltrado
peribronquial, edema
submucoso y obstrucción de la
pequeña VA → atelectasias y
atrapamiento aéreo
Cuadro Clínico
• Primera Fase: Infección
de VAS (tos, rinorrea,
compromiso conjuntival y
OMA), fiebre escasa.
• Segunda fase: dificultad
respiratoria obstrucción
variable y progresiva,
estertores, hipoxemia,
acidosis respiratoria o
mixta.
• Tercera fase:
normalización gradual de
la hipoxemia, tos catarral
CLAVE E
Preguntas de Pediatría
21. ¿Cuál es el principal desencadenante de la
crisis asmática?
A. Cuerpos extraños
B. Hipertrofia adenoidea
C. Ingesta de corticoides
D. Sinusitis
E. Infeccion viral
Factores exacerbantes
• Infecciones respiratorias
• Alérgenos
• Ejercicios e hiperventilación
• Cambios de clima
• Dioxido de azufre
• Comida, aditivos, drogas
CLAVE E
Preguntas de Pediatría
22. ¿Cuál es el fármaco que se utiliza para el
tratamiento de los episodios agudos de asma?
A. Albuterol
B. Ketotifeno
C. Bromuro de Ipratropio
D. Cromoglicato sódico
E. Teofilina
Manejar exacerbaciones del asma
• Terapia primaria para exacerbaciones:
• Administración repetida de β2-agonistas
inhalados de acción corta
• Introducción temprana de corticoides
sistémicos
• Suplemento de oxígeno
• Monitorear respuesta tempranamente con
medidas repetidas de función pulmonar
CLAVE A
Preguntas de Pediatría
23. En un paciente con asma aguda, que en la valoración
inicial se encontró VEF ≥ de 40%, la conducta seria:
A. Beta 2 agonistas de acción corta 5 dosis + bromuro de
ipatropio
B. Beta 2 agonistas de acción corta 3 dosis + corticoides
orales sistémicos
C. Beta 2 agonistas de acción prolongada 1 dosis +
corticoides inhalados
D. Beta 2 agonistas de acción prolongada + inhibidores de
lucotrienos
E. Metilxantinas endovenosa + corticoides parenterales
CLAVE B
Preguntas de Pediatría
24. En el asma ¿Cuál de los siguientes
medicamentos es de control a largo plazo?
A. Agonistas B2 de acción rápida
B. Anticolinergicos
C. Corticoides parenterales
D. Antihistaminicos
E. Modificadores de leucotrienos
CLAVE E
Preguntas de Pediatría
25. Niño asmático de 4 años, con asma nivel 3
¿Cuál es el tratamiento indicado?
A. Corticoides inhalados dosis media
B. Corticoides inhalados dosis alta + teofilina
C. Corticoides inhalados dosis media + agonistas
B2 de acción prolongada
D. Corticoides inhalados dosis baja + agonistas
B2 acción corta
E. Corticoides inhalados dosis alta + corticoides
sistémicos.
CLAVE A
Preguntas de Pediatría
26. En el tratamiento del asma persistente grave
se indica…
A. corticoide sistémico.
B. antihistamínico.
C. antileucotrieno.
D. cromoglicato sódico.
E. teofilina.
CLAVE A
Preguntas de Pediatría
27. Paciente de 8 años con hematoquezia y
prolapso rectal, que en el examen prequirúrgico de
rutina encuentran Hb 11.8, leucocitos 10,000,
eosinófilos 10%. ¿En qué parásito sospecha?
A. Ancylostoma duodenale
B. Ascaris lumbricoides
C. Trichocephalo dispar
D. Enterobius vermicularis
E. Fasciola hepática
Trichuris trichiura

Trichuriasis
*Nematodo
*Habitat: Intestino grueso
* Comensal
*En grandes cantidades : Patogeno
*Tricocefalosis, tricuriasis
Trichuris trichiura
Morfologia
*Tamaño: 30 a 40 mm
* Extremo anterior muy fino.
*Extremo posterior grueso, en los machos curvo.
*Huevo de 40 a 50 micras. Forma de barril, polos con tapones
mucosos
Trichuris trichiura
Ciclo vital:
Hábitat : Intestino grueso.
Vía de infección :Oral
Forma Infectante:
Huevo larvado
Dos a cuatro semanas en el
medio ambiente para larvar
Trichuris trichiura
Epidemiología:

*Similar a la de Ascaris en condiciones de suelo, clima y


humedad.
*Tasa de infeccion similar a la de Ascaris
*Cosmopolita
*Prevalencia alta, carga parasitaria baja
Trichuris trichiura
Patogenia:

*Relacionada con la cantidad de parásitos existentes.


*Una pequeña carga parasitaria no daría síntomas
*Los casos masivos se asocian con niños con trastornos
nutritivos graves.
* Adultos hematófagos.
Extremo anterior del nematodo
se introduce en la mucosa.
*Aumenta el peristaltismo por
la irritación de los plexos nerviosos
Trichuris trichiura
Sintomatología:
*Generalmente asintomático
*Trichuriasis masiva:
Síntomas digestivos: Crisis disentericas
Heces mucosanguinolentas
Pujo, Tenesmo
Dolores abdominales
Prolapso rectal
Sintomas generales: (Producto de la anemia)
Palidez
Astenia
Anorexia
Retardo de crecimiento
Trichuris trichiura
Diagnostico:
*En las infecciones leves el hemograma es normal.
*En las infecciones masivas presenta:
Anemia hipocroma microcitica
Eosinofilia elevada
* Parasitologico:
- Examen seriado de deposiciones MTM, PAFS
Trichuris trichiura
Tratamiento
*Mebendazol
*Albendazol

Control:
Colectivo
*Impedir la contaminación humana fecal del medio
*Servicio de alcantarillados adecuados
Individual
*Lavado de frutas y verduras antes de consumir
*Lavado riguroso de manos
CLAVE C
Preguntas de Pediatría
28. Niña de 12 años con dolor epigástrico urente a
predominio nocturno, distensión abdominal y
deposiciones sueltas claras post prandial
intermitente ¿En qué parásito sospecha?
A. Giardia lamblia
B. Trichocephalo dispar
C. Fasciola hepática
D. Echinococcus granulosis
E. Ascaris lumbricoides
GIARDIASIS
• AGENTE. Giardia lamblia
• CICLO VITAL
• Ingestión de quistes (+25), resisten
jugos gástricos, en duodeno se
rompen liberando trofozoitos, se
adosan a mucosa intestinal
causando reacción inflamatoria y
deficiencia en la absorción de
diversos nutrientes.
• Hábitat
– Duodeno
Giardiasis
Cuadro Clínico

• Diarrea
• Nauseas
• Vómito
• Anorexia
• Dolor epigástrico
• Bruxismo
• Meteorismo
• Retardo en el
crecimiento
CLAVE A
Preguntas de Pediatría
29. La Giardia lamblia habita en:
A. Duodeno y Yeyuno proximal
B. Yeyuno distal e ileon
C. Colon
D. Recto
E. Estómago
Giardiasis
Diagnostico

• CPS Seriado
• Examen en fresco de
líquido duodenal
• Cápsula de Beal
CLAVE A
Preguntas de Pediatría
30. Niño de 5 años con diarrea crónica, esteatorrea,
dolor abdominal, pérdida de peso, malabsorción de
grasa y vitaminas liposolubles. ¿Cuál sería su
indicación terapéutica?
A. Metronidazol 10 mg/kg/día por 10 días
B. Metronidazol 15 mg/kg/día por 5 días
C. Albendazol 15 mg/kg/día por 10 días
D. Albendazol 10 mg/Kg/día por 5 días
E. Cotrimoxazol 10 mg/Kg/día por 7 días
Giardiasis
Tratamiento

• Metronidazol 15mg/kg/día c/8hrs. X 5 a 7 días


• Furazolidona 6mg/kg/día c/6hrs. X 7 a 10 días

Red book pediatria


CLAVE B
Preguntas de Pediatría
31. ¿Cuál de los siguientes parásitos no realiza
ciclo de Looss?
A. Ascaris Lumbricoides
B. Estrongiloides Stercolaris
C. Ancylostoma Duodenale
D. Trichurios trichura
E. Necátor Americano
Ciclo de Loos
• Recorrido que realizan las larvas
de ciertos nemátodos que les
permite madurar en el interior de
su hospedero.
• Ascaris lumbricoides,
Strongyloides stercoralis y las
Uncinarias
• Las larvas infectantes pasan
desde el intestino a la circulación
mesentérica, de ahí van al hígado,
luego al corazón derecho, migran
hacia el pulmón y llegan a los
alvéolos, tráquea y finalmente
son deglutidos para alcanzar su
estado adulto en el lumen
intestinal.
El síndrome de Loeffler
• Loeffler en 1932 describe un síndrome caracterizado
por infiltrado pulmonar transitorio, asintomáticos y con
eosinofilia asociado a parasitosis
• Ascaris lumbricoides, Strongyloides stercoralis,
Ancylostoma duodenale y Necator americanus
• Neumonía eosinofílica en el contexto de una infección
donde las manifestaciones clínicas más frecuentes son :
tos seca, disnea, sibilantes, malestar retroesternal y
fiebre baja
• Pronóstico favorable y resolución en unas 3 semanas,
no se ha demostrado que el tratamiento influya en la
evolución
CLAVE D
Preguntas de Pediatría
32. Acerca de las caracteristicas
epidemiológicas de la taenia solium es cierto:
a) El ser humano es hospedero definitivo
b) El cerdo es hospedero definitivo
c) La tenia adulta vive a nivel del ileo
d) En la mayor parte de los casos se tiene el
diagnostico definitivo
e) El perro es el hospedero definitivo
Ciclo de vida de la Tenia solium
CLAVE A
Preguntas de Pediatría
33. La Neurocisticercosis se debe a la ingesta de:
A. Huevos de Taenia sollium
B. Huevos de Taenia saginata
C. Larvas de Taenia sollium
D. Larvas de Taenia saginata
E. Larvas y huevos de Taenia saginata
Ciclo de vida de la Tenia solium
Teniasis
• Generalmente se
acepta que la
cisticercosis se adquiere
al ingerir alimentos, o
líquidos contaminados
con huevecillos
Cisticercosis

• Al comer carne de puerco


contaminada con cisticercos
se adquiere la Teniasis
Teniasis
Cuadro clínico

• Perdida de peso
• Anorexia
• Palidez
• Dolor abdominal
• Distensión abdominal
• Presencia de
proglótidos en la heces
Cisticercosis
Cuadro Clínico
• SNC
– Crisis convulsivas
– Edema cerebral
– Cráneo hipertensivo
• Músculo
• Ojos
– Disminución de la
agudeza visual
– Amaurosis
• Corazón
• Hígado
Cisticercosis
Dxs.
• Tamizado de heces
– Proglótidos
• BH
• Estudio de LCR
• Tac y/o RM de
cráneo o área
afectada
Tratamiento
• Teniasis • Cisticercosis
– Niclosamida • Hospitalización
• 1 gr. Dosis única – Prazicuantel 50
masticadas
mg/Kg. Día por 14 días
– Dexametasona 0.5 mg x
– Prazicuantel 10 kg x dosis
mg/Kg. Dosis Única
– Sintomático
• Crisis convulsivas
• Cefaleas
• Cráneo hipertensivo
Albendazol
• FARMACOCINÉTICA Y FARMACODINAMIA:
– Después de la administración oral, albendazol es absorbido en poca cantidad
(menos del 5%). A la dosis de 6.6 mg/kg de albendazol, la concentración en
plasma de su principal metabolito, el sulfóxido de albendazol, alcanza un
máximo de 0.25 a 0.30 mcg/ml después de aproximadamente dos horas. La
vida media del sulfóxido de albendazol en el plasma es de aproximadamente
8.5 horas.

• El metabolito es eliminado esencialmente por la orina.

• Ejerce su efecto antihelmíntico, inhibiendo la polimerización de la tubulina


, por tanto, depleta los niveles energéticos hasta que estos llegan a ser
insuficientes para la sobrevivencia de los parásitos. De este modo,
albendazol inicialmente inmoviliza y después mata a los helmintos
susceptibles.
Mebendazol
• FARMACOCINÉTICA Y FARMACODINAMIA:
– Al administrarse por la vía oral virtualmente no se absorbe. Su acción
se lleva a cabo en la luz del intestino. Se elimina casi totalmente por
las heces entre las 24 y 48 h después de su administración. Se ha
detectado entre un 5 a 10% de la dosis ingerida, en el torrente
urinario.
• El Mebendazol bloquea la captación de glucosa por los
nematodos, provocando una depleción del glucógeno y del
adenosíntrifosfato necesarios para la supervivencia de los
parásitos, lo que ocasiona su muerte lenta. Afortunadamente
no interfiere en el metabolismo de los carbohidratos que se
lleva a cabo en el ser humano.
Preguntas de Pediatría
34. Niña de 18 meses, con hiporexia, prurito anal
intenso sobre todo nocturno que incluso
interrumpe el sueño. ¿Cuál es la dosis de
albendazol para un tratamiento completo?
A. 100mg 1 sola dosis y repetir a los 30 días
B. 100mg 2 dosis y repetir a los 8 días
C. 200mg 1 sola dosis y repetir a las 2 semanas
D. 400mg 1 sola dosis
E. 400mg 1 sola dosis y repetir a los 10 días
OXIURIASIS
• AGENTE. Enterobius
vermicularis
• CICLO VITAL
– Ingestión de huevos que se
depositan en duodeno
liberando larvas en los
pliegues de yeyuno e íleon.
• Se completa
aproximadamente en 2
meses.
OXIURIASIS
Contagio

• Auto infección por ano-


mano-boca
• Contacto directo al
saludarse
• Inhalación de huevos
en el polvo
• Migración retrograda a
través del ano
OXIURIASIS
Cuadro Clínico

• Prurito anal
• Insomnio
• Irritabilidad
• Vulvitis
• Dolor abdominal
• Nausea
• Bruxismo .
Oxiuriasis

• CPS seriado
• Graham
Oxiuriasis
Tratamiento

• Pamoato de Pirantel solución oral


– 11 mg/Kg de producto base una vez
• Mebendazol solución oral
– 100 mg. cada 12 hr. por 3 días
– Repetir en 2 semanas
• Albendazol Solución oral
– 400 mg. Dosis única (mayores de 2 años)
– 200 mg. Dosis única (menores de 2 años)
– Repetir en 2 semanas
CLAVE C
Preguntas de Pediatría
35. Escolar con cuadro de obstrucción intestinal. Al
estudio por imágenes se observa ovillo de Ascaris.
¿Cuál es el tratamiento previo a la intervención?
A. Pamoato de pirantel
B. Praziquantrel
C. Citrato de piperacina
D. Albendazol
E. Tiabendazol
ASCARIASIS
Áscaris Lumbricoides

• AGENTE.
– Áscaris lumbricoides
• CICLO VITAL
Huevos embrionados
Larvas migrans
Larvas Rabditoide
Forma adulta
Ciclo de vida del Ascaris Lumbricoides
ASCARIASIS
Contagio

• Ingestión de huevos
embrionados
– Ano-Mano-Boca
– Alimentados contaminados
con tierra.
– Geofagia
• Suelos húmedos y
sombreados
• Más común en
preescolares y escolares.
ASCARIASIS
• En intestino delgado son liberadas
las larvas las cuales se conocen como
larvas migrans
• Estas penetran la mucosa intestinal
hasta alcanzar los vasos
mesentéricos y vía porta llegan hasta
la membrana alveolo capilar
atravesándola
• Ascendiendo el árbol respiratorio (
Larvas Rabditoide)
• Llegando hasta la glotis para
posteriormente ser deglutidas y
transformarse en la forma adulta
capaz de eliminar huevos
ASCARIASIS
Cuadro Clínico

• Síndrome de Löefler.
• Eosinofilia
• Bronconeumonía
• Ascaridiasis
• Anorexia
• Palidez
• Mal estado general
• Perdida de peso
• Geofagia
• Bruxiismo
• Obstrucción intestinal
• Expulsión parásito
– Boca
– Ano
ASCARIASIS
Diagnóstico

• Coproparasitoscopico
seriado
– Búsqueda de huevos
• Rx abdomen.
– Imagen en madeja de
estambre.
– Obstrucción Intestinal
ASCARIASIS
Tratamiento
• Mebendazol solución
oral
– 100 Mg. cada 12 hr. por
3 días

• Albendazol Solución
oral
– 400 mg. Dosis única
Suboclusión por Ascaris
• La administración de pamoato de pirantel o
mebendazol precipita la obstrucción intestinal,
ya que su mecanismo de acción causa parálisis
espástica en los gusanos.
• El citrato de piperazina produce una parálisis
flácida y se indica 75 mg/kg/día (máx 2.5
g/día) VO por 2 días.
• En los casos de obstrucción grave es necesaria
la intervención quirúrgica
CLAVE C
Preguntas de Pediatría
36. ¿Cuál es el fármaco de elección para el
tratamiento de la estrongyloidiasis?
A. Ivermectina
B. Secnidazol
C. Triclabendazol
D. Albendazol
e. Nitazoxanida
Strongyloides stercolaris
Biología

• Nematodo muy pequeños 2mm x 50µ


• Ciclo vital con alternancia de generaciones de
vida libre y parasitaria
• Via de infeccion :Cutanea
• Del huevo emergen larvas rabditiformes
• Forma infectante: larva filariforme
• Es el único capaz de reproducirse en el ser
humano y permanecer en el en forma
indefinida
Ciclo Vital (1)
•(A) Larvas filariformes penetran
por la piel
•(B) Larvas llegan al corazón
derecho y luego a los pulmones
•(C, D) Larvas llegan a la traquea y
luego son deglutidas
•(E) Adultos en el duodeno
•(F) Hembras ponen huevos en las
criptas duodenales
•(G) De los huevos emergen
larvas
•(H) Huevos y larvas salen con las
deposiciones
Formas clinicas
-Auto infección : Es la capacidad de este nematodo de iniciar un nuevo
ciclo sin salir al exterior
-Hiperinfección. Es el sobrecrecimiento de parásitos con el
consecuente aumento en la maduración de larvas rabditiformes a
filariformes, lo que puede ocurrir a lo largo de los sitios por donde
realiza su ciclo de vida. Generalmente se asocia con algún tipo de
inmunodeficiencia
Diseminada. Se refiere a la invasión de la larva filariforme de sitios
fuera del tracto gastrointestinal o el pulmón.
Larvas migrantes cutaneas
-Hiperinfeccion en un
paciente
inmunodeprimido
-Intenso edema pulmonar
bilateral
-Se le considera un cuadro
inflamatorio y
hemorragico
- Este cuadro es una de las
principales enfermedades
pulmonares en individuos
inmunodeprimidos
Adultos en el duodeno
-En individuos inmuno
deprimidos o en tratamientos
con corticoesteroides se
pueden producir intensos
casos de infeccion
generalizada con reacciones
inflamatorias y
granulomatosas
Diagnostico
• Paciente de zona endémica.
• Manifestaciones cutáneas, dolor abdominal y
diarrea.
• Eosinofilia elevada
• Hallazgo de larvas rabditiformes en
deposiciones es difícil
• Elisa. Sensibilidad 85%- 90%
Tratamiento
• Ivermectina
- 200 mcg/Kg/día en monodosis por 1 a 2 días
• Tiabendazol
- 50 mg/Kg/día en 2 dosis por 2 días
Prevención
• Evitar contaminacion.
• Mejorar la calidad del agua de bebida.
• Estimular el uso de calzado
CLAVE A
Preguntas de Pediatría
37. Escolar de 12 años procedente de Junin,
presenta ictericia, dolor en hipocondrio derecho de
3 meses de evolución, hemoglobina de 3 grs/dL.
¿Cuál es el tratamiento?
A. Bitionol
B. Prazicuantel
C. Pamoato de pirantel
D. Triclabendazol
E. Tinidazol
FASCIOLASIS Fasciola hepatica

SUCESIÓN DE FASES LARVARIAS

Miracidio  redia    cercaria   metacercaria

176
Fasciola hepatica.
H.Definitivo: Ovidos, bóvidos, conejos, hombre .
H. Intermediario: gasterópodo
Limnaea, Succinea, Fossaria, Stagnicola, Bulinus.
Ubicación: Conductos biliares.
Ciclo de vida de la Fasciola
Cuadro clínico
• La fase aguda, puede durar entre 4 a 6 meses y se
presenta clínicamente con fiebre, hepatomegalia y
eosinofilia.
• A veces se complica y puede presentarse como:
hematoma subcapsular hepático, ruptura hepática,
múltiples abscesos hepáticos, granuloma hepático,
anemia severa, ictericia, colangitis, colecistitis,
pancreatitis, masa hepática, derrames pleurales, y
eventualmente fibrosis hepática
• Fase Crónica: Dolor abdominal, diarrea y pérdida de
peso.
Tratamiento
• Triclabendazol 10 mg/Kg dosis única VO
• Bitionol 30 a 50 mg/Kg en días alternos por 10
a 15 dosis
CLAVE D
Preguntas de Pediatría
38. Paciente procedente de la selva, con dolor
torácico, disnea y expectoración hemoptoica. En la
radiografía de tórax se observan imágenes pulmonares
cambiantes, en el hemograma eosinófilos 8% y al examen
de heces se encuentran huevos redondeados con capa
externa mamelonada. ¿Cuál es el parásito causal
probable?
A. Ascaris lumbricoides
B. Tricocéfalo dispar
C. Ancylostoma duodenale
D. Necator americanus
E. Strongyloides stercolaris
ASCARIASIS
Cuadro Clínico

• Síndrome de Löefler.
• Eosinofilia
• Bronconeumonía
• Ascaridiasis
• Anorexia
• Palidez
• Mal estado general
• Perdida de peso
• Geofagia
• Bruxiismo
• Obstrucción intestinal
• Expulsión parásito
– Boca
– Ano
ASCARIASIS
Diagnóstico

• Coproparasitoscopico
seriado
– Búsqueda de huevos
• Rx abdomen.
– Imagen en madeja de
estambre.
– Obstrucción Intestinal
CLAVE A
Preguntas de Pediatría
39. Niña de 10 meses que inicia con fiebre alta, tos
persistente, rinorrea, congestión ocular al tercer día
aparecen lesiones exantemáticas maculopapulares
confluentes en la cabeza y se van extendiendo al
resto del cuerpo. ¿Cuál es el diagnóstico probable?
A. Sarampión
B. Rubeola
C. Varicela
D. Eritema infeccioso
E. Roseola infantil
SARAMPIÓN
• P de incubación de 10 días.
• Prodrómico 2- 4 días:
Triple catarro.
• Enantema : Manchas de Koplik
• Exantema máculo papular eritematoso:
rojo oscuro- cefalocaudal.
• Tumefacción facial
• Duración 1 semana

• Fiebre persistente.
SARAMPION

1. Sospecha clínica:
Triple catarro: conjuntival,
nasal, bronquial.
Enantema bucal
característico.
Exantema morbiliforme,
cefalocaudal.
Fiebre alta mientras
progresa la erupción.

2. IgM antisarampionosa en
sangre en período agudo.
 Generalización rápida
y duración efímera.
 Cefalocaudal
 Linfadenopatía
generalizada.
 Síntomas generales
leves o ausentes.
SARAMPIÓN
SARAMPIÓN
Complicaciones del sarampión
• Neumonía: justifica el 60% de las muertes
causadas por sarampión.
• La incidencia de complicaciones es mayor en
sujetos desnutridos y en las edades extremas
de la vida.
• En los pacientes con neumonía asociada a
sarampión es frecuente una infección
bacteriana secundaria.
Complicaciones del sarampión
• En niños que carecen de inmunidad mediada
por linfocitos T, aparece una neumonía de
células gigantes sin exantema.
Complicaciones del sarampión
• Encefalitis: es una de las complicaciones mas
temidas.
• Puede llegar a afectar a un 0.5% de los afectados,
suele ser mortal en un 15% de los casos.
• Acostumbra manifestarse entre 7 y 10 días después
del inicio del sarampión.
• Esta se debe a reacciones inmunopatologicas, se
asocia aun proceso de desmielinizacion de las
neuronas y afecta con una mayor frecuencia a niños
mayores y adultos.
CLAVE E
Preguntas de Pediatría
40. Niño de 2 años, presenta fiebre, leve congestión
nasal y anorexia. Al examen: micro adenopatías
cervicales y occipitales. A las 72 horas de enfermedad
cae la fiebre y aparece erupción maculopapular que se
inició en tronco y se extiende al cuello, cara y se
generaliza a las 24 horas, desapareciendo al tercer día.
¿Cuál es el diagnóstico MÁS probable?:
A. Varicela.
B. Sarampión.
C. Exantema súbito.
D. Rubeóla.
E. Escarlatina.
EXANTEMA SUBITO
(6TA.ENFERMEDAD)
• Niños 6 meses – 3 años

• Agente etiológico: Herpesvirus Tipo 6 (Herpesviridae)

• Contagio:
• Secreciones respiratorias
• Contacto interhumano intimo

Incubación: 5-15 días


EXANTEMA SUBITO
(6TA.ENFERMEDAD)
CLINICA
• Fiebre alta (3-5 días)

• RASH: maculopapular diseminado en cuello, tronco y cara que


aparece luego de la fiebre de forma centrífuga

• Asociado o no a otros síntomas tales como faringitis, rinitis,


congestión timpánica, adenomegalias, conjuntivitis, etc.
EXANTEMA SUBITO
(6TA.ENFERMEDAD)

• COMPLICACIONES: compromiso SNC, meningitis, encefalitis,


CONVULSIONES FEBRILES.

• DIAGNOSTICO:
• Clínico
• Laboratorio : Serologías específicas IgG e IgM

• TRATAMIENTO:
• De sostén
Imágenes Sexta Enfermedad
CLAVE C
Preguntas de Pediatría
41. Niño de 8 años que presenta inicialmente febrícula y
exantema eritematoso localizado en las mejillas que
rápidamente se extiende al tronco y a las extremidades
proximales, posteriormente presenta un aclaramiento
central de las lesiones maculares dando un aspecto
reticulado. Se debe sospechar en:
A. Eritema Infeccioso.
B. Exantema súbito.
C. Escarlatina.
D. Rubéola.
E. Sarampión
Eritema Infeccioso

 Exantema máculo papular bifásico :


facial y luego universal.

 Acentuación en extremidades y
nalgas.

 Evoluciona a patrón reticular

 Síntomas y signos agregados leves o


ausentes.
Eritema infeccioso

• Tiempo de incubación 4 a 14 días


• Contagio por secreciones
respiratorias
• Exantema simétrico, macular
máculo papular, eritematoso y
evanescente, con palidez central
Patrón reticular o de encaje
• Intensidad variable en el día
• Reactivación.
Eritema infeccioso

 Asociado a infecciones
por Parvovirus B19.

 Causa crisis de aplasia


aguda en pacientes con
hemoglobinopatías y
anemia crónica.

 Infección durante la
gestación puede causar
muerte fetal.
 Afecta : Niños 1- 4 años.
 Frecuente en mujeres.
 Brote estacional.
 Exantema :
morbiliforme
escarlatiniforme
eczematoso.
 Afecta flexuras: axilas- ingle
 Avanza a tronco y áreas
vecinas.
 Adenopatía regional
 Prurito leve 60%
ERITEMA INFECCIOSO
(5TA. ENFERMEDAD)
• CLINICA:
• Exantema simétrico macular o maculopapular festoneado en
placas.
• 2 fases: Signo de la cachetada (1-2 días)
Progresión a extremidades, nalgas y tronco
(1-2 semanas)

• COMPLICACIONES: anemia, aplasia eritroidea

• DIAGNOSTICO: clínico

TRATAMIENTO: de sostén
CLAVE A
Preguntas de Pediatría
42. Niña de dos años, que acude a su consulta por fiebre alta de 5 días
de evolución. En la exploración: labios eritematosos, lengua muy roja,
ganglios cervicales, conjuntivitis y exantema maculopapular en tronco,
edema en manos y pies. Respecto al diagnóstico más probable del
caso, señale usted cuál de las siguientes afirmaciones es CIERTA:

A. La descamación en dedo de guante es típica de la fase aguda de la


enfermedad.
B. En la fase subaguda de la enfermedad, dentro de los datos destaca
fundamentalmente la trombocitopenia.
C. La administración de AAS en altas dosis previene la aparición de
aneurismas coronarios.
D. La afectación cardiaca ocurre en un 10-40% de los casos.
E. Los esteroides son unos de los pilares fundamentales del
tratamiento
Enfermedad de Kawasaki
• 1era causa de cardiopatía
adquirida.
• Más frecuente en Japón y EUA.
Epidemiología • Más frecuente en varones,
menores de 5 años.

• Infecciosa : Parvovirus B19,


Epstein Barr, Herpes,
chlamydia pneumoniae.
Etiología • Predominio en invierno y
primavera
Enfermedad de Kawasaki
• Patogenia:
– Predisposición genética
• Gen CCR5
Toxinas
bacterianas LT, polimorfo Citoquinas
(superantígeno nucleares, proinflamat
s) macrófagos orias

Edema de
musculo
liso y IgA
endotelio
Enfermedad de Kawasaki
– El vaso pierde su integridad estructural, se debilita,
dilatación.
– Trombo, aneurismas, estenosis.
– Arterias de mediano calibre= Principalmente coronarias
Enfermedad de kawasaki

• Disociación por edema 6-8 días


• Compromiso de pared 10 días
• Daño severo, aneurisma 12 días
• Persistencia de inflamación 25 días
• Desaparición de células inflamatorias 40 días
Enfermedad de Kawasaki
• Clínica:
– Fiebre por mas de 5 días (no responde a tx, 1-2
sem= enf. coronaria) y 4 de los siguientes criterios:
• Inyección conjuntival bilateral no exudativa. 98%
Enfermedad de Kawasaki
• Boca y faringe eritematosas, lengua
aframbuesada, labios rojos y fisurados. 96%
Enfermedad de Kawasaki
• Rash generalizado, polimórfico, confluyente,
no vesiculoso. 86%
Enfermedad de Kawasaki
• Induración de manos y pies con eritema de
palmas y plantas, descamación periungueal.
66%
Enfermedad de Kawasaki
• Surcos transversos en uñas de los dedos.
(Líneas de Beau) 2-3 meses del inicio de los
síntomas.
Enfermedad de Kawasaki
• Adenopatías cervicales, no supurativas, al
menos un ganglio superior a 1.5 cm de
diámetro. 44%
Enfermedad de Kawasaki
• Clínica:
– Si no se cumplen los criterios pero mediante ECO
o coronografía se demuestran aneurismas
coronarios = KAWASAKI ATÍPICO
– Menores de un año y mayores de 8 años
– 1era y 4ta semana
Enfermedad de Kawasaki
Other clinical and Laboratory findings
Cardiovascular findings Musculoskeletal system Central nervous system

Congestive heart failure, Arthritis, arthralgia Extreme irritability


myocarditis, pericarditis,
valvular regurgitation

Coronary artery Gastrointestinal tract Aseptic meningitis


abnormalities
Aneurysms of medium- Diarrhea, vomiting, Sensorineural hearing
size noncoronary arteries abdominal pain loss

Raynaud phenomenon Hepatic dysfunction Genitourinary system

Peripheral gangrene Hydrops of gallbladder Urethritis/meatitis


Enfermedad de Kawasaki
• Alteraciones cardiacas:

• Miocarditis: Taquicardia. 50%


• Disminución en la fx ventricular
• Pericarditis
• Aneurismas de arterias coronarias (≥8 mm
diametro interno, gigantes)
• Riesgo de ruptura, trombosis o estenosis, e IM
• Regurgitación valvular importante.
Enfermedad de Kawasaki
• Diagnostico
• Fase aguda se encuentra:
– Leucocitosis >20.000 en el 50%
– >30.000 en el 15% de los casos.
– VSG acelerada, frecuentemente
>100 mm (1ª hora).
– Plaquetas normales inicialmente y después de la 2ª
semana de enfermedad suben a 1-2 millones/mm3.
Enfermedad de Kawasaki
• Proteína C reactiva positiva.
• Hipoalbuminemia.
• Aumento de TGO, TGP, fosfatasa alcalina y bilirrubina.
• Piuria estéril con ligera proteinuria y microhematuria.
• Pleocitosis moderada en LCR. Cultivos negativos. Posibles
signos de anemia hemolítica
Enfermedad de Kawasaki
• El ecocardiograma en la fase aguda puede
• mostrar alteraciones (disminución de la
• fracción de eyección y aneurismas a partir
• de la 1ª semana y sobre todo a las 3-4
• semanas).
Enfermedad de Kawasaki
• Evolución
– Fase aguda febril (1-2 semanas).

Complicaciones: aneurismas arteriales, valvulitis y


miocarditis.
Enfermedad de Kawasaki
• Evolución
• b) Fase subaguda. (4ta semana)
• Cede la fiebre, el exantema y la adenopatía.
• Persiste la inyección conjuntival y la irritabilidad.
• Inicia la descamación furfurácea perineal y en
dedos en láminas, junto con artritis/artralgias.
• Infartos de miocardio o iniciarse
• Aneurismas coronarios.
• Trombocitosis.
Enfermedad de kawasaki

• Fase subaguda:
• Leucocitosis, neutrofilia
• Elevación de sedimentación eritrocitaria
• Elevación de proteína C reactiva
• Anemia
• Hipoalbuminemia, hiponatremia
• Elevación de transaminasas
Enfermedad de Kawasaki
c) Fase de convalecencia. (6-8 sem)

• Las manifestaciones clínicas desaparecen


• Los reactantes de fase aguda y el resto de las alteraciones de
laboratorio se normalizan.
• Los aneurismas pueden resolverse o persistir y dejar una
disfunción cardiaca o presentar infartos de miocardio.
Enfermedad de Kawasaki
• Tratamiento

• Aspirina. Dosis inicial 80-100 mg/kg/día


• en 3-4 dosis, durante 2 semanas o hasta 48
• horas después de que el paciente quede
• afebril.

• Anomalías de las arterias coronaria, a la se añadirá


dipiridamol a 3-5 mg/kg/día.
Enfermedad de Kawasaki
• Gammaglobulina intravenosa (IGIV)
• 2 g/kg, en dosis única, lentamente.
• Eficaz en los 10 primeros días de la enfermedad.

• Efectos secundarios, la IGIV puede


• provocar insuficiencia cardiaca por aumento de
volemia y meningitis aséptica.
CLAVE E
Preguntas de Pediatría
43. Paciente de 5 años de edad, desde hace dos días
presenta fiebre., malestar general, disfagia, hiporexia,
vómitos. Al examen: faringe congestiva, exantema de
color rojo punteado papuloso generalizado, más intenso
en pliegues de flexión, excepto en la zona peribucal ¿Cuál
es el tratamiento de elección?
A. Clorfenamina
B. Penicilina
C. Ibuprofeno
D. Loratadina
E. Aciclovir
ESCARLATINA
• Agente etiológico:
Estreptococo B hemolítico del GRUPO A

• Contagio:
• Secreciones respiratorias
• 24Hs. Posteriores de iniciado el tratamiento

• Incubación: 2-5 días


ESCARLATINA
• RASH:
• Eritematoso micropapular en tronco y extremidades
• Aspecto: “papel de lija”
• Descamación furfurácea (a los 7-10 días)
• Signo de Pastia (más acentuado en pliegues)
• Facie de Filatow (surcos nasogenianos libres)
• ANGINA: congestión, eritema, exudado, adenopatias
submaxilares
Lengua aframbuesada
Se asocia con vómitos, cefalea y dolor
abdominal
ESCARLATINA
• COMPLICACIONES:
• Fiebre reumática
• Glomerulonefritis
• Absceso periamigdalino y retrofaringeo

• DIAGNOSTICO:
• Clínico
• Laboratorio : CULTIVO DE FAUCES +
TRATAMIENTO:
Penicilina 50.000UI/Kg/dia. VO. c/8-12 Hs
Amoxicilina 40-50 mg/Kg/dia VO. c/8Hs.
(durante 10 días)
ESCARLATINA
CLAVE B
Preguntas de Pediatría
44. Lactante de 12 meses, presenta febrícula, eritema
faríngeo, manchas color rosa en el paladar blando,
adenopatías cervicales y retroauriculares dolorosas. Al
día siguiente, exantema maculopapular confluente que
empieza en cara y se extiende a todo el cuerpo. El
probable diagnóstico es:
A. Rubeola.
B. Sarampión.
C. Exantema súbito.
D. Infección por CMV.
E. Varicela.
RUBEOLA
Rubéola
 Exantema de rápida
evolución- se apaga
mientras se disemina.
 Duración 2- 3 días

 Enantema bucal-ocular.
Signo de Forschheimer.

 Afectación articular,
hematopoyética y
neurológica.
CLAVE A
Preguntas de Pediatría
45. Una niña de 3 años previamente sana, bien vacunada,
comenzó hace 3 días con unas lesiones cutáneas
pequeñas, maculosas y rojas, que evolucionaron a
vesículas claras en el tórax, cara, muslos y brazos. En la
exploración destacan algunas costras y pústulas, 3
pequeñas úlceras en la lengua y paladar, y dos lesiones
similares en los labios menores. ¿Cuál es su diagnóstico?:
A. Primoinfección herpética.
B. Síndrome pie-mano-boca.
C. Erupción variceloforme de Kaposi.
D. Varicela.
E. Herpangina.
VARICELA
• Mayor frecuencia en menores de 10 años
• Prevalece en invierno y primavera
• Agente etiológico: Varicela Zoster (Herpes Virus)
• Contagio:
• Secreciones respiratorias
• Contacto persona a persona
• Contacto con exudado de la lesión
• Incubación: 10-21 días
VARICELA
CLINICA
• Inicio : fiebre, rinitis, malestar general
• RASH:
• Maculo-papulo-vesiculo-costroso
• Distribucion cefalocaudal
• Centrifugo
• Pruriginoso
• Dura aprox. 7 días
• No respeta cuero cabelludo
Compromete mucosas (oral, genital, conjuntival)
VARICELA
• DIAGNOSTICO:
• Clínico
• Laboratorio : Cultivo o PCR
Serologías específicas IgM e IgG

• COMPLICACIONES:
• Sobreinfección bacteriana
• Complicaciones SNC (meningitis, encefalitis)
• Sindrome de Reye (varicela asociada con aspirina)
• Miocarditis, artritis, hepatitis, etc.
VARICELA
Tratamiento:
• Menores de 12 años: sintomático
• Mayores de 12 años: Aciclovir 80mg/kg/día. VO. durante 5
días. Acorta período de fiebre y disminuye lesiones)
• Inmunocomprometidos: Aciclovir 40 mg/kg/día. EV. durante
5-8 días.

• PREVENCION:
• Vacuna anti varicela: entre los 12 meses y 11años (dosis
única)
Gamaglobulina: antes de las 96hs de
exposición en todo paciente
inmunocomprometidos
Imágenes Varicela
CLAVE D
Preguntas de Pediatría
46. Niña de 7 años presenta dolor en garganta,
fiebre 39°C, dos días después, exantema
eritematoso en todo el cuerpo, que respeta el surco
nasogeniano, además lengua fresa. El diagnóstico
MÁS probable es:
A. Eritema infeccioso.
B. Infección por adenovirus.
C. Escarlatina.
D. Difteria.
E. Exantema súbito.
Imágenes Escarlatina
ESCARLATINA
CLAVE C
Preguntas de Pediatría
47. Niño de 9 años que acude a Urgencias por presentar tumefacción cervical
bilateral, sin fiebre ni otros síntomas. En la exploración física presenta: BEG,
febrícula, palpación de varias adenopatías dolorosas de 1,5 cm, bilaterales en
región submandibular, y un exudado blanco-amarillento sobre amígdalas
hipertróficas e hiperémicas. En el frotis nasofaríngeo aparece cultivo positivo
para estreptococo beta hemolítico del grupo A. Se instaura tratamiento con
amoxicilina v.o., a pesar de lo cual la paciente regresa al 5to día por
persistencia del cuadro, fatiga intensa, exantema maculopapuloso en tronco y
extremidades, visceromegalia y fiebre de 39°C. ¿Cuál es el diagnóstico más
probable?:
A. Herpangina.
B. Escarlatina.
C. Sarampión.
D. Rubéola.
E. Mononucleosis infecciosa.
¿Faringitis viral o bacteriana?
• Clínica: Dolor de garganta,
fiebre alta, malestar, fatiga,
dolor abdominal, nausea,
cefalea
• Examen físico: Faringitis
exudativa, adenopatía
cervical dolorosa, hepato-
esplenomegalia.

Mononucleosis Infecciosa
DIAGNOSTICO
MONONUCLEOSIS INFECCIOSA: TRIADA
- CRITERIO CLINICO: FIEBRE-FARINGITIS-ADENO-

ESPLENOMEGALIA
- CRITERIO HEMATOLOGICO:LINFOCITOSIS-LINFOC.
ATIPICOS
- CRITERIO SEROLOGICO : ANTICUERPOS
HETEROFILOS
CLAVE E
Preguntas de Pediatría
48. La panencefalitis esclerosante subaguda o
encefatitis de Dawson tiene como etiología al
virus de:
a Rubeola
b Sarampión
c Influenza
d Varicela
e Hepatitis B
Secuelas del Sarampión

• Panencefalitis esclerosante
subaguda (PEES): es una secuela
neurológica muy tardía de
extraordinaria gravedad que
afecta aproximadamente a 7 de
cada millón de pacientes.
• Esta enfermedad aparece cuando
un virus defectuoso sobrevive en
el cerebro y actúa como un virus
lento.
Secuelas del sarampión
(PEES)
• El virus se puede multiplicar y diseminarse de una célula a
otra pero no se libera.
• La PEES es mas prevalente en niños que se infectaron antes de
los 2 años y se manifiesta (la PEES) aprox. 7 años después de
la enfermedad.
• El paciente presenta cambios de personalidad,
comportamiento y memoria, seguidos de contracciones
mioclónicas, ceguera y espasmos.
• Los pacientes suelen presentar niveles elevados de
anticuerpos frente al virus del sarampión en sangre y
LCR.
CLAVE B
Preguntas de Pediatría
49. ¿Cuál de las siguientes características es
más útil para establecer la diferencia entre
rubeola y exantema súbito?
a Edad de presentación
b Respuesta al tratamiento
c Adenopatías
d Tipo de exantema
e Hemograma
CLAVE D
Preguntas de Pediatría
50. Para el diagnóstico de infección urinaria en
escolares. ¿Cuál es el método más adecuado de
recolección de la muestra?:
A. Bolsa colectora
B. Cateterización
C. Chorro medio
D. Sondaje permanente
E. Punción infrapúbica
Datos analíticos en orina
Test Sensibilidad % Especificidad %

Esterasa leucocitaria 83 (67 – 94) 78 (64 – 92)


( EL)
Nitritos (N) 53 (15 – 82) 98 (90 – 100)

EL o N 93 (90 – 100) 72 (58 – 91)

Leucocituria 73 (32 – 100) 81 (45 – 98)

Bacteriuria (B) 81 (16 – 99) 83 (11 – 100)

EL o N o B 99,8 (99 – 100) 70 (60 – 92)

Subcommittee on Urinary Tract Infection (AAP). Pediatrics 1999


Diagnóstico definitivo de ITU
• Sondaje uretral: Urocultivo + si > 10.000 col/ml
• ITU segura si coexisten leucocituria y urocultivo +.
• Urocultivo positivo sin leucocituria puede ser bacteriuria
asintomática (2 %)
• ITU: VPP de la leucocituria para Urocultivo + es de 75%
• Leucocituria inespecífica: 9% de lactantes con fiebre
Métodos de recolección de orina
Métodos de recolección de orina
AAP 2011 Clinical Practice Guideline

Diagnosis and Management of the Initial UTI in Febrile


Infants and Children, 2 to 24 Months*

*Guideline: Pediatrics. 2011;128(3):595–610


Technical report: Pediatrics. 2011;128(3):e749–e770
What’s New in This Revision
1. Diagnosis
– Abnormal urinalysis as well as positive culture
– Positive culture = ≥50,000 colony-forming units (cfu)/mL
– Assessment of likelihood of UTI
2. Treatment: Oral as effective as parenteral
3. Imaging: Voiding cystourethrography (VCUG) not
recommended routinely after first febrile UTI
4. Follow-up: Emphasis on urine testing with subsequent
febrile illnesses
CLAVE C
Preguntas de Pediatría
51. Lactante de 13 meses que acude a urgencias por fiebre de hasta 39ºC de
48h de evolución sin otra sintomatología asociada. Exploración por órganos y
aparatos sin hallazgos significativos, destacando buen estado general. Usted
le iba a dar de alta, pero el pediatra de guardia le pide examen de orina y un
urocultivo por sondaje. En la orina destaca leucocituria ++, hematuria + y
nitritos ++ y en el Gram de orina se observan bacilos Gram negativos. En el
hemograma no existe leucocitosis y la proteína C reactiva es de 50 mg/l. El
pediatra le dice que no es preciso que el niño sea hospitalizado y que le paute
un antibiótico oral. Señale el tratamiento antibiótico empírico menos
adecuado en este caso:
A. Amoxicilina
B. Amoxicilina-clavulánico
C. Cefuroxima-axetilo
D. Cefalexina
E. Cefixima
Tratamiento empírico
Edad condición Primera elección Segunda elección Observaciones
clínica

RN y lactante Cefalosporina de Aminoglicosidos Cobertura anti


menor 3ra parenteral + + ampicilina enterococco
Ampicilina
Lactante mayor Cefalosporina de Aminoglicosidos Rotar a
en MEG 3ra parenteral cefalosporina VO
de 2da o 1ra
Lactante mayor Cefalosporina VO Cefalosporina VO Ajustar según
en BEG de 1ra de 2da cultivo

Paciente Cefalosporina de Aminoglicosidos Cobertura anti


inmunosuprimido 3ra parenteral + + ampicilina enterococco
Ampicilina
Recomendaciones
• Una vez que el niño esta estable y afebril se
cambia a via oral. La AAP recomienda
tratamiento por 7 a 14 dias en niños de 2
meses a 2 años.
• En ITU complicada el tratamiento debe ser por
2 semanas
Gauthier M, Chevalier I, Sterescu A, et al. Treatment of urinary tract infections among febrile young children with daily
intravenous antibiotic therapy at a day treatment center. Pediatrics 2004;114(4):e469–76.
CLAVE A
Preguntas de Pediatría
52. ¿Cuál es el método de imagen más utilizado
y menos traumático para el estudio de las
infecciones urinarias recurrentes en niñas?
A. Ecografía renovesical
B. Urografía
C. Cistouretrografía
D. Cistografía isotópica
E. Gamagrafía
CLAVE A
Preguntas de Pediatría
53. Es importante diagnosticar precozmente la
infección del tracto urinario alto en el lactante
para prevenir:
A. Estreñimiento
B. Vaciado incompleto de la vejiga
C. Cicatrices renales
D. Desnutrición
E. Enuresis nocturna
Factores de riesgo que predisponen a daño
renal
• Edad:
Mayor riesgo antes del año
Menor riesgo después de los cuatro años
• Retraso en el inicio del tratamiento
No debe ser mayor de tres días
• Reflujo Vesicoureteral
Asociado a malformaciones
• Otros factores: gérmenes, inmunidad

Saieh C. Rev Med CLC 1995;6:81-3


CLAVE C
Preguntas de Pediatría
54. El reflujo vesicoureteral es causa de:
A. Infección urinaria a repetición
B. Hipotensión
C. Obstrucción ureteral
D. Síndrome nefrótico
E. Poliquistosis renal
Grados de Reflujo Vesicoureteral
Cicatrices renales comparado con el grado de
reflujo
Grade of reflujo % de cicatrices
1 5
2 6
3 17
4 25
5 50

ªModified from Skoog SJ, Belman AB, Majd M.A nonsurgical approach to the
management of primary vesicoureteral reflux. J Urol 1987; 138: 441.
CLAVE A
Preguntas de Pediatría
55. ¿Cuál de las siguientes afirmaciones referentes
al reflujo vesico-ureteral grado I en el niño es
CORRECTA?:
A. Se diagnostica por ecografía.
B. Se debe tratar con profilaxis antibiótica.
C. Tiene tendencia a desaparecer.
D. Produce pielectasia renal.
E. Debe corregirse quirúrgicamente de forma
precoz.
Profilaxis en ITU: ¿dar o no dar?
Montini G, Hewitt I. Pediatr Nephrol. 2009 May 15

• No esta indicada la profilaxis en el primer


episodio de ITU
• No esta indicada la profilaxis en RVU grado I o
II si no presenta recurrencia
• El porcentaje de reinfecciones es mayor en el
RVU grado III 24/87 vs 11/137
CLAVE C
Preguntas de Pediatría
56. ¿Cuál de los siguientes casos es indicación de
prueba de tuberculina en niños?
A. Extranjeros de países desarrollados
B. Contacto familiar con personas con
tuberculosis
C. Al inicio de la etapa pre-escolar
D. Historia de TBC en el paciente
E. Pacientes con tratamiento de 1mes de
corticoides
CLAVE B
Preguntas de Pediatría
57. La induración de 5 milímetros en la prueba
de tuberculosis se considera positiva en:
A. Linfoma
B. Viaje a zonas de alta prevalencia
C. Nacimiento en países de alta prevalencia
D. Tratamiento inmunosupresor
E. Insuficiencia renal crónica
Valores positivos de la PPD en niños
Induración ≥ 5 mm
Niños en contacto cercano con un caso confirmado de
tuberculosis:
• Sin tratamiento
• Iniciando tratamiento aún con sintomatología
• Con tratamiento no supervisado
Niños sospechosos de padecer tuberculosis.
• Radiografía de tórax con cambios radiológicos que sean
compatibles con tuberculosis activa o previamente activa.
• Evidencia clínica de tuberculosis.
Niños con infección por VIH y niños inmunosuprimidos por
tratamiento con esteroides u otras drogas
American Academy of Pediatrics, Pediatric Tuberculosis Collaborative Group. Targeted tuberculin skin testing and treatment of
latent tuberculosis infection in children and adolescents. Pediatrics 2004 Oct; 114(4):1175-1201.
Valores positivos de la PPD en niños
Induración ≥ 10 mm
Niños con alto riesgo de enfermedad diseminada:
• Niños menores de cuatro años de edad.
• Factores médicos de riesgo, como: enfermedad de Hodgkin,
linfoma, diabetes mellitus, insuficiencia renal crónica,desnutrición.
Niños con mayor riesgo de exposición a casos de TB:
• Nacidos o hijos de padres nativos de regiones con alta prevalencia
de TB: Latinoamérica, Asia, África.
• Viajes a regiones con alta prevalencia.
• Niños con exposición frecuente a adultos infectados por VIH,
usuarios de drogas intravenosas, indigentes, residentes de asilos,
personas encarceladas o inmigrantes.
Induración ≥ 15 mm
Niños de cuatro o más años sin factores de riesgo agregados.

American Academy of Pediatrics, Pediatric Tuberculosis Collaborative Group. Targeted tuberculin skin testing and treatment of
latent tuberculosis infection in children and adolescents. Pediatrics 2004 Oct; 114(4):1175-1201.
CLAVE D
Preguntas de Pediatría
58. ¿En cuál de las siguientes formas clínicas de
la tuberculosis se añade corticoides al
tratamiento específico?
A. Meníngea
B. Renal
C. Pulmonar
D. Ganglionar
E. Genital
Indicación de corticoides en TBC
• Esta indicado en TB miliar y TB con inflamación
de serosas (TB pleural, intestinal y/o peritoneal,
pericárdica y meníngea).
• La dosis recomendada es 1-1.5 mglKgldía de
prednisona (o su equivalente) por 2-4 semanas.
• Está indicado su uso en RAFA moderada a severa
en dosis: 1-2 mglKg./día de prednisona (o su
equivalente) durante el tiempo necesario para
controlar el evento adverso.
CLAVE A
Preguntas de Pediatría
59. Madre de 18 años de edad refiere que ayer fue evaluado por su
médico por presentar hemoptisis y BK +++. Solicita evaluación para su
bebé de 18 días de edad con lactancia materna exclusiva. Peso 3.5Kgr.
Deposiciones semi líquidas amarillas de 7 a 10 veces/día, recibió BCG,
actualmente asintomático. Nacido de parto eutócico peso 3.4Kg, Apgar
9, 10. ¿Cuál es su diagnóstico y conducta?
A. Infección: PPD, BK, observación hasta resultado
B. Contacto: BK, por sonda orogástrica (3), radiografía de tórax.
C. Enfermedad suspender lactancia matema, tratamiento con 3
fármacos, BK por sonda orogástrica (3)
D. Contacto: INH 5mgIKg día x 3m. luego PPD, si es positivo continua
tratamiento hasta 6 meses
E. Contacto descartar infeccíón, PPD, radiografía de tórax
Sospechar TB
• Fiebre > 7 días
• Falla de crecimiento
• Neumonía que no responde a antibióticos
• Neumonía con cavitación
• Adenopatía hiliar
• Derrame pleural sin neumonía
Elementos Diagnósticos

• Exposición a persona enferma (BAAR


positivo)
• Evidencia de infección (PPD positivo)
• Cuadro clínico - radiológico
• Identificación del microorganismo por
baciloscopía (BAAR) y/o cultivo
(Mycobacterium tuberculosis complex)
PPD positivo

• Es el mejor indicador de infección


tuberculosa
• Falsos positivos por reacción cruzada
(BCG, otras micobacterias)
• Falsos negativos en lactantes, ancianos,
inmunodeficiencias
• Inmunización con BCG no invalida
• Valor predictivo positivo en relación a
prevalencia
PPD según grupo etáreo
110 113
120 107

100

Casos 80

60

40
10 13 7
20
0 0 0
0
0-4 mm Grupo A Grupo B Grupo C
5-9 mm Grupo etáreo
> 10 mm

Aching J. Tesis bachiller. 2000


Cuadro clínico-radiológico
• Cuando hay adenopatía hiliar es común que no
haya síntomas o solamente fiebre.
• Pérdida de peso y tos crónica son comunes en
enfermedad cavitaria.
• Radiografía de tórax puede mostrar adenopatía
hiliar, derrame pleural, colapso/consolidación,
enfermedad cavitaria o miliar.
• Enfermedad extrapulmonar no es rara en niños.
Patogenia
Inhalación Invasión de alvéolos

Foco primario

Hipersensibilidad tipo IV Activación de macrófagos

Diseminación linfática Diseminación hematógena

Progresión del foco primario Infección latente

Neumonía (cavitación), Enfermedad miliar,


adenopatía hiliar, derrame enf cavitaria o
pleural enf extrapulmonar
CLAVE B
Preguntas de Pediatría
60. Niño de un año de edad cuya madre acaba de ser diagnosticada de
tuberculosis pulmonar bacilífera, habiendo iniciado tratamiento correcto. El
niño está asintomático con una exploración normal, prueba de tuberculina
(PT) negativa y Rx tórax normal. ¿Cuál sería la actitud a adoptar en el niño?

A. Solo separarlo de la madre hasta que ésta concluya su tratamiento (al


menos 6 meses)
B. Realizar una TC de alta resolución pulmonar, y si es normal, repetir la PT a
las 8-12 semanas.
C. Quimiprofilaxis con isoniacida (INH) hasta que la baciloscopia de la madre
sea negativa.
D. Quimioprofilaxis con INH 6-9 meses. Nueva PT y Rx tórax al finalizarlo, para
decidir si concluir o no, el tratamiento.
E. Quimioprofilaxis con INH, 12 semanas. Repetir PT. Si positiva, realizar Rx
tórax para decidir si hay o no enfermedad.
CLAVE E
Preguntas de Pediatría
61. Niño de cuatro años, cuya madre BK positivo recibe
tratamiento con el esquema I. Tiene PPD 12 mm. BK
negativo y la radiografía muestra infiltrado parenquimal
en base derecha. ¿Cuál es su diagnóstico y tratamiento?

A. TBC pulmonar y esquema I


B. Contacto TBC y quimioprofilaxis
C. Primoinfección y quimioprofilaxis
D. TBC pulmonar y esquema II
E. Contacto TBC y prueba terapéutica
CLAVE A
Preguntas de Pediatría
62. En los niños menores de 2 años con
tuberculosis pulmonar primaria el hallazgo
radiológico más frecuente es:
A. Adenopatias hiliares o mediastinicas
B. Derrame pleural
C. Atelectasia lobar o segmentaria
D. Neumotórax
E. Infiltrado miconodular en lóbulo superior
CLAVE A
Preguntas de Pediatría
63. Ante un niño de 7 meses con fiebre e irritabilidad,
fontanela abombada y un estudio de líquido cefalorraquídeo
con 110 células/mm3 (75% linfocitos), proteínas 120 mg/dl y
glucosa 28 mg/dl (glucemia sérica 89 mg/dl), ¿cuál es la
sospecha diagnóstica más razonable?

A. Meningitis vírica
B. Meningitis bacteriana
C. Meningitis tuberculosa
D. Síndrome mononucleósico
E. Síndrome de Guillain-Barré
MENINGITIS TUBERCULOSA
• Forma muy grave de tuberculosis con alta tasa de mortalidad y
secuelas.
• El diagnóstico temprano y el tratamiento precoz resultan vitales.
PATOGENIA
• Por diseminación hematógena, los bacilos alcanzan los plexos coroideos e invaden el
espacio subaracnoideo inflamando las meninges.
• Por contigüidad, a partir de un tuberculoma cerebral que fistuliza al espacio
subaracnoideo.
• Muy raramente, por contigüidad a partir de un hueso del cráneo (TBC ósea).
ANATOMÍA PATOLOGICA
• Exudado fibrinoso, espeso localizado preferentemente en la base del cráneo
dificultando la circulación del LCR.
MENINGITIS TUBERCULOSA
DIAGNÓSTICO
CUADRO CLÍNICO
• Astenia, apatía, constipación, cambios de carácter, fiebre y cefaleas progresivas que
aumentan de intensidad, vómitos explosivos, fotofobia, midriasis, parálisis del 3º y 4º
par, convulsiones, hemiparesia, síndrome confusional progresivo, grave compromiso de
la conciencia, letargia, delirio, estupor, coma y muerte.
• Clásicamente se han descripto 3 etapas progresivas:
(Consejo Británico de Investigación Médica)
• Etapa 1:Predominan manifestaciones generales. Paciente conciente y lúcido. No hay
signos neurológicos.
• Etapa 2: paciente confuso, con signos neurológicos y de hipertensión endocraneana
(cefalea intensa, fiebre, vómitos centrales, compromiso del sensorio, convulsiones,
hiperestesia cutánea, midriasis, rigidez de nuca, signo de Körning y Brudzinsky.
• Etapa 3: Grave compromiso de la conciencia, letargia, estupor, delirio, hemiplejía o
paraplejía, coma y muerte.
MENINGITIS TUBERCULOSA
DIAGNÓSTICO
• En presencia de un cuadro febril con síndrome meníngeo se debe realizar Punción
Lumbar, previo examen de fondo de ojo, para descartar signos de Hipertensión
Endocraneana.
Características del LCR:
• Cristal de Roca. Cuando el cuadro lleva cierto tiempo es levemente amarillento o
xantocrómico por depósitos de fibrina. Se puede observar un sobrenadante
reticulofibrinoso en “Velo de novia”.
• Tensión Aumentada.
• Aumento de Proteínas
• Aumento de la celularidad, al inicio aumento PMN luego linfocitosis.
• Aumento de albúmina.
• Disminución de glucosa ( menor 40mg/dl) y cloruros.
• Examen bacteriológico (Directo y Cultivo). El Cultivo aumenta el rendimiento y permite
realizar prueba de sensibilidad.
MENINGITIS TUBERCULOSA
DIAGNÓSTICO
DIAGNOSTICO POR IMÁGENES
• Rx Tx: Compatibles con TBC pulmonar y frecuentemente siembras miliares.
• TAC cerebro: Calcificación central rodeada de una zona hiperdensa con contraste EV.
rodeado de edema perilesional (Signo de Welchman)
• RMN cerebral.
DIAGNOSTICO DIFERENCIAL
• Con meningitis de otra etiología:
Virales: Enterovirus ,mixovirus, adenovirus, citomegalovirus, sarampión
Micoticas: Criptococosis, especialmente en pacientes con SIDA.
Parasitarias: Toxoplasmosis especialmente en pacientes con SIDA.

• Sífilis.
• Neoplasias.
MENINGITIS TUBERCULOSA
PRONÓSTICO
• Depende fundamentalmente de la rapidez del tratamiento, el que debe instaurarse ante
la menor sospecha de la enfermedad.
• La mitad de los pacientes que curan, quedan con secuelas neurológicas (ceguera,
sordera, hidrocefalia, parálisis, convulsiones, alteraciones mentales, déficit motores y/ o
sensitivos)
TRATAMIENTO
• Fase intensiva o de ataque: 2 meses H-R-Z-E diaria
• Fase consolidación: 10 meses H-R diaria
• La estreptomicina solo alcanza LCR cuando las meninges están inflamadas.
• En la meningitis tuberculosa se ha demostrado que el uso inicial de corticoides,
disminuye la mortalidad, no así las secuelas de la enfermedad. Meprednisona 0.5 a1
mg/k/d durante 30días y luego reducción gradual.
Resonancia magnética nuclear
(RMN) con múltiples
tuberculomas, junto con zonas de
isquemia en ganglios basales,
mesencéfalo y protuberancia
B. Vila, C. Martín, P. Leal, M.L. Rodríguez
Hospital Provincial de la Misericordia. Toledo.
España.
CLAVE C
Preguntas de Pediatría
64. Paciente de 15 años con tos, hemoptisis y baja de
peso que es atendida con los siguientes resultados: BK ++,
cultivo en proceso; prueba rápida de sensibilidad:
resistente a rifampicina e isoniacida, sensible a etambutol
y pirazinamida. El diagnóstico bajo criterios de la
estrategia sanitaria es un caso de TBC pulmonar…
A. Resistente
B. Multi drogoresistente
C. Extremadamente resistente
D. Totalmente resistente
E. Sensible
TBC MDR
• Es causada por el organismo (M. tuberculosis)
que es resistente, por lo menos, a los dos
fármacos de primera línea más eficaces
• Isoniacida
• Rifampicina
TBC MDR
• Resistencia primaria es la transmisión de
organismos drogorresistentes de persona a
persona
• Resistencia secundaria se desarrolla durante
el tratamiento de TB
• No se trató al paciente con el esquema de
tratamiento apropiado
• El paciente no sigue el esquema de
tratamiento tal como se le indico
CLAVE B
Preguntas de Pediatría
65. ¿Cuál es el diagnóstico presuntivo frente a un
paciente de 8 años de edad, que presenta un
absceso frío, con destrucción de la piel
suprayacente y ganglio cervical ulcerado?
A. Goma sifilítico
B. Esporotricosis
C. Hidradenitis supurativa
D. Escrófula
E. Forúnculo
TUBERCULOSIS GANGLIONAR
• Frecuente forma de presentación extrapulmonar.
• Según localización puede afectar:
1.-Ganglios periféricos o superficiales: los ganglios cervicales son los más afectados ,
(Adenitis cervical: Escrófula) especialmente los submaxilares y supraclaviculares. Pueden
afectarse también los ganglios axilares e inguinales.
2.-Adenopatías internas: especialmente traqueobronquiales, secundarios a una
primoinfección tuberculosa a nivel pulmonar. Frecuente en niños y pacientes VIH.
3.-Ganglionar generalizada: Por diseminación hematógena, típica en pacientes HIV.
• Hacer diagnostico diferencial con las adenitis provocadas por micobacterias atípicas
(escrofulaceum; avium), que ingresan al organismo por otras vías distintas a la pulmonar. En
los niños se puede aislar el complejo avium de las adenitis superficiales; en los adultos, en el
90% de los casos el responsable es Micobacterium tuberculosis.
Por lo tanto, Cultivar las muestras de biopsia o por PAAF (punción aspiración con aguja fina)
TUBERCULOSIS GANGLIONAR
PATOGENIA
• Diseminación por vía linfática, generalmente posterior a una TBC primaria
(frecuente en niños y jóvenes). También en adultos HIV.
DIAGNÓSTICO
1.- Cuadro Clínico
Comienzo insidioso. Adenopatías duras, indoloras, no adheridas, la piel que la
cubre presenta signos de inflamación. Pueden fistulizarse y drenar material
caseoso. Por lo general no producen síntomas sistémicos.
2.- Laboratorio
Examen bacteriológico, histopatológico y citológico de las muestras obtenidas
por PAAF o biopsia.
3.-PCR para identificar Micobacterias atípicas.
4.-TAC: útil en TBC ganglionar interna o generalizada.
TUBERCULOSIS GANGLIONAR
PRONÓSTICO
• Favorable. Pueden aparecer nuevas adenopatías, o fistulizarse durante el tratamiento.
• Algunas veces el tamaño retrotrae muy lentamente, un 5-10% mantiene el mismo
tamaño al final de tratamiento, comportándose como adenopatías residuales, sin que
indique mala evolución.
TRATAMIENTO
• Fase de ataque: 2 meses H-R-Z-E
• Fase de consolidación: 7 meses H-R
• Quirúrgico en aquellos casos complicados en que existe compresión severa del árbol
traqueobronquial o de algún órgano intraabdominal.
DIAGNÓSTICO DIFERENCIAL
• Micobacteriosis atípicas
• Infecciones por gérmenes comunes, micosis, parásitos.
• Sarcoidosis. Linfomas. Enfermedad Hodking. Metástasis neoplásicas.
CLAVE D
Preguntas de Pediatría
66. ¿A qué edad en meses se considera que un
lactante saludable alimentado con lactancia
materna tiene riesgo de ferropenia?
A. 4
B. 2
C. 3
D. 12
E. 8
CLAVE A
Preguntas de Pediatría
67. Paciente de 15 años, con antecedentes de enfermedad de Crohn y
resección de íleon terminal hace 3 años, presenta intensa astenia y palidez
cutáneo-mucosa. En la bioquímica destaca LDH 2730 UI/L y bilirrubina 1,7
mg/dL. En el hemograma muestra 2,9 x 109 leucocitos/L con recuento
diferencial normal, hemoglobina 7,5 g/dL, hematocrito 22%, VCM 135 fl y 105
x 109 plaquetas/L. El recuento de reticulocitos es 1% y en el frotis de sangre
periférica se observan abundantes neutrófilos hipersegmentados. ¿Cuál sería
su juicio diagnóstico más probable y su actitud terapéutica inicial?:
A. Anemia de probable origen inflamatorio. Transfundir concentrado de
hematíes.
B. Anemia megaloblástica por déficit de vitamina B12. Iniciar vitamina
B12 parenteral y ácido fólico oral.
C. Anemia hemolítica. Iniciar corticoides.
D. Síndrome mielodisplásico. Transfundir con-centrado de hematíes.
E. Anemia megaloblástica por déficit de ácido fólico. Iniciar ácido fólico
oral.
ANEMIAS POR ALTERACION EN LA SINTESIS DEL DNA

Deficiencia de vitamina B12 y Acido Fólico

Características comunes:
Se presentan alteraciones en todas las lineas sanguíneas y en las
células de rápido recambio:
Glóbulos rojos, blancos, plaquetas, células de la mucosa bucal vaginal, intestinal.
Células con núcleo grande y citoplasma abundante
ANEMIAS MEGALOBLASTICAS POR CARENCIA DE VITAMINA B12

APORTE INADECUADO : Vegeteriano estricto

ABSORCION DEFECTUOSA: Ausencia de Factor Intrínseco (Gastrectomía, Gastritis atrófica)


Defecto Absoción Intestinal (síndrome de mala absorcion,
medicamentos, enfermedad pancreática
DESTRUCCION BACTERIANA: Síndrome de asa ciega, diverticulosis

CONSUMO COMPETITIVO : Infectación con botriocéfalo

DEFICIT EN EL TRANSPORTE: Déficit congénito de transcobalamina


CLAVE B
Preguntas de Pediatría
68. La proteína que almacena el hierro
intracelular es
A. Rubredoxina
B. Hemoglobina
C. Transferrina
D. Superoxidodismutasa
E. Ferritina
Metabolismo del Hierro
CLAVE E
Preguntas de Pediatría
69. Varón de 4 años de edad. Hace un mes, luego de un
cuadro respiratorio alto presenta dolor en miembros
inferiores y rodillas, anorexia, fatiga y febrícula
intermitente. Acude por presentar epistaxis. Al examen se
observa palidez, equimosis, petequias, adenopatías
sésiles no dolorosas, hepatoesplenomegalia. ¿Cuál es el
diagnóstico más probable?
A. Leucemia Linfoblástica Aguda
B. Mononucleosis Infecciosa
C. Purpura Trombocitopénica Idiopática
D. Purpura de Henoch
E. Hemofilia A
Epidemiología
• Es el cáncer infantil más común
• El subtipo más frecuente es la Leucemia
Linfoblástica aguda (LLA)
• La LLA ocurre en el 75 a 80% de todos los
casos de Leucemia en niños
• La Leucemia Mieloide aguda (LMA) ocurre en
el 20% de los casos
Hallazgos clínicos
• Son frecuentes los signos y síntomas de
anemia y trombocitopenia
• La neutropenia puede lleva a infecciones
severas
• El dolor de huesos y las artralgias son
causadas por la infiltración leucémica
• Es más común en LLA que en LMA
• Es especialmente severa en niños pequeños
Hallazgos clínicos
• La LMA es la responsable del 30% de las
muertes por leucemia en niños
• Peor respuesta al tratamiento quimioterápico
• Mayor número de complicaciones
hemorrágicas al diagnóstico
• Necesidad de tratamientos más agresivos
como el trasplante de progenitores
hematopoyéticos
Hallazgos clínicos
• El compromiso extramedular en LLA ocurre
en: hígado, bazo, timo y nódulos linfoides
• El compromiso extramedular en LMA ocurre
en: piel, encías, cabeza y cuello
• La infiltración del SNC puede encontrarse
tanto en LLA y LMA
Diagnóstico diferencial
• Un inicio agudo con sangrado, petequias y equimosis
puede sugerir PTI
• Tanto la leucemia como la anemia aplásica pueden
presentarse con pancitopenia y falla de la médula
ósea
• Infecciones virales como mononucleosis pueden
simular leucemia
• El dolor óseo, artralgias y artritis pueden observarse
en colagenopatías como ARJ, fiebre reumática y
también en osteomielitis
Tratamiento
• Los pacientes con LLA reciben una
relativamente corta fase de inducción
remisión
• Seguida de una terapia de consolidación y
luego un tratamiento largo de continuación
• Todos los pacientes requieren manejo del
compromiso subclínico del SNC intratecal
Tratamiento
• El manejo de la LMA incluye terapia de
remisión inducción y consolidación
• Las terapias posteriores difieren ampliamente
• El transplante autólogo de células madre no
es usualmente recomendado
Indicaciones de transplante alogénico
• LLA con cromosoma Filadelfia
• Recaída hematológica temprana
• LLA de células T con pobre respuesta
temprana
• LMA mejora la sobrevida global
Pronóstico
• La sobrevida actual a los 5 años es cerca del
80%
• El cromosoma filadelfia es un marcador de
mal pronóstico
CLAVE A
Preguntas de Pediatría
70. ¿Cuál es el tumor sólido más frecuente en
niños?
A. Leucemia linfoblastica aguda
B. Linfoma
C. Astrocitoma
D. Retinoblastoma
E. Leucemia mieloide aguda
Oncología Pediatrica
• Frecuencia de Cáncer en menores de 15 años
Tasa / 1,000,000 Sobrevida a 5 años
– Todo el Cáncer Infantil 136.4 75%
– Leucemia 38.2 78%
– Tumores del S.N.C. 26.3 65%
– Linfoma 13.5 83%
– Neuroblastoma 10.2 66%
– Tumor de Wilms 8.8 89%
– Rabdomiosarcoma 8.3 65%
Oncología Pediátrica
• Durante el 1er año de vida predominan los tumores
de origen embrionario por desarrollarse durante el
proceso de diferenciación celular tales como:

– Neuroblastoma
– Tumor de Wilms (Nefroblastoma)
– Retinoblastoma
– Rabdomiosarcoma
– Meduloblastoma
Oncología Pediátrica
• Entre los 2 y 5 años de edad, predominan los
tumores de origen embrional como:

– Leucemias Agudas
– Linfoma No Hodgkin
– Gliomas
Oncología Pediatrica
• Alrededor de la pubertad predominan tumores
relacionados con el crecimiento y el desarrollo
sexual

– Tumores malignos de Hueso


– Linfoma de Hodgkin
– Cáncer gonadal (testicular y ovárico)
– Cáncer de tiroides
Incidencia por edad de distintos tipos de Cáncer

Incidence of the most common types of cancer in children by age.


The cumulative incidence is shown as a dashed line.
(Courtesy of Archie Bleyer, MD.)
Tumores del SNC en niños
Introducción
• Ocupan el segundo lugar en frecuencia, después de
la leucemia
• Son las formas más frecuentes de las masas
tumorales sólidas
• Rara vez metastatizan fuera del SNC
• Resultan localmente invasivos
• Las estructuras afectadas no se regeneran
adecuadamente
• Muchas veces las formas benignas, por la región
donde asientan, evolucionan hacia la malignidad
Introducción
• En Estados Unidos de Norteamérica se informan
anualmente 1500 casos nuevos
• Estos neoplasias (generalmente astrocitomas,
meduloblastomas y ependimonas) se localizan en la
región infratentorial
• El grado de malignidad dependerá de su crecimiento,
obstrucción que produzcan, ubicación en el tronco
cerebral, tálamo e hipotálamo, así como propagación
Introducción
• En los niños predominan los tumores infratentoriales
(55%) sobre los supratentoriales (45%), excepto en
lactantes
• En Europa y Norteamérica, predominan el
astrocitoma cerebeloso y el meduloblastoma
• En África y Japón, hay una mayor incidencia de
craneofaringiomas y tumores de la región pineal
• El ependimoma es más frecuente en la India que en
cualquier otro país
Hallazgos clínicos
• La cefalea es un síntoma prácticamente constante en
los tumores cerebrales infantiles
• En el lactante, puede manifestarse como irritabilidad
o llanto inconsolable
• Cefalea durante el sueño, historia corta (< 6 meses)
de dolor de cabeza, asociación con confusión o
vómitos, ausencia de síntomas visuales y presencia
de signos neurológicos anormales
Hallazgos clínicos
• La cefalea puede ir acompañada de vómitos,
que suelen ser matutinos o nocturnos
• Puede ir o no precedidos de náuseas
• Los síntomas y signos de focalidad son más
evidentes cuanto mayor es la edad del niño
Hallazgos clínicos
• La hemiparesia, hipertonía e hiperreflexia son las
manifestaciones más frecuentes en los tumores
supratentoriales
• En los infratentoriales, los síntomas y signos más
habituales son el mareo, los vómitos, la dipoplia, la
ataxia y el nistagmus
• Tortícolis puede ser la primera manifestación de un
tumor de fosa posterior
• En los tumores de tronco, puede haber parálisis de
pares craneales
Tratamiento
• Existen diversas opciones de tratamiento
• Fundamentalmente son tres: cirugía, quimioterapia y
radioterapia
• La radioterapia local está indicada en tumores malignos, en
propensos a recidivar, en éxeresis incompletas, e incluso en
tumores de bajo grado inextirpables
• Se emplea la quimioterapia en tumores como el
meduloblastoma de alto riesgo y los astrocitomas de alto
grado
• La quimioterapia puede seguir a la radioterapia,o puede
precederla
CLAVE C
Preguntas de Pediatría
71. Niño de 4 años de edad que consulta por aparición a lo largo de los últimos tres
días, de lesiones cutáneas en piernas y glúteos. Sus padres referían que su pediatra le
había diagnosticado una infección de vías respiratorias superiores 10 días antes. En las
últimas 12 horas presenta dolor abdominal intenso tipo cólico, y ha realizado dos
deposiciones diarreicas. Afebril. No pérdida de peso. A la exploración presenta
numerosas petequias y lesiones purpúricas palpables de predominio en glúteos y
extremidades inferiores. Buen estado general aunque tiene dolor abdominal intenso,
La palpación abdominal es difícil de valorar por dolor difuso. No visceromegalias. No
otros hallazgos de interés a la exploración. ¿Cuál de los siguientes datos NO apoya su
sospecha diagnóstica?

A. Artritis de rodillas y tobillos


B. Hematuria
C. Plaquetopenia
D. Sangre oculta en heces
E. Edema escrotal
EPIDEMIOLOGÍA
• Más frecuente en niños de 3-10 a
• 4 meses- adultos
• 50% < 5a 75% < 10a
• Incidencia: 13.5- 18/100,000
• Varones/mujeres: 1.5- 2/1
• Aumenta en invierno y primavera
• Es la vasculitis más común en niños
ETIOLOGÍA
• Desconocida, se relaciona con infecciones
respiratorias recientes
• Agentes infecciosos: estreptococo beta hemolítico
grupo A, adenovirus, hepatitis B, HIV, parvovirus B19,
micoplasma, herpes virus, helicobacter pilori,
toxocara canis, etc.
• Medicamentos: penicilinas, sulfonamidas, alopurinol,
propiltiouracilo, quinidina, etc.
• Otros: exposición al frío, comidas, mordedura de
insectos, etc.
DIAGNÓSTICO
• 1990 ACR: dos o más criterios
1. Edad menor de 20a
2. Púrpura palpable
3. Dolor abdominal o sangrado GI
4. BX con evidencia de granulocitos en la pared de
arteriolas o venulas
• Consenso de Chapel Hill: presencia de vasculitis en
pequeños vasos + depósitos de IgA
EXANTEMA PURPURICO
• Es palpable de 2 a 10mm de diámetro
• Urticaria generalizada  maculopapulas
eritematosas  petequias  púrpura en placas
• Rojo púrpura  amarillo  púrpura marrón 
desaparición
• Afecta primariamente extremidades inferiores y
nalgas. Cara en niños menores de 2a
• Lesiones necróticas o bulosas < 5%
REVISIÓN DE CASOS DE SAULSBURY
COMPROMISO GASTROINTESTINAL
• 2/3 de los niños afectados, 1/3 con sangrado
gastrointestinal
• SX: dolor abdominal tipo cólico, nauseas
vómitos, hemorragia intestinal,
intususcepción, pancreatitis e hidrops
vesicular
• Los síntomas GI pueden preceder al rash en 14
– 36%
ARTRITIS
• Ocurre en 60-84% de los casos
• Es transitorio, afecta generalmente tobillos y
rodillas, no deja deformidades
• Saulsbury reporta que 37% presentan además
compromiso de manos y muñecas
• 24% fue la manifestación inicial
COMPROMISO RENAL
• Raro que preceda al rash
• Ocurre en el 50% de niños mayores pero es
serio en 10%, 2-5% IRC
• En el 80% el compromiso empieza las
primeras 4 semanas
• Hematuria microscópica  sind. nefritico/
sind. nefrotico  IRC
• Peor pronóstico si el compromiso glomerular >
del 50%
• Todo paciente debe tener examen de orina,
urea, creatinina, PA de inicio
• Repetir el examen de orina cada semana
mientras la enfermedad esté activa
• Si no hay evidencia de nefritis un examen
mensual hasta 3 meses después de lo
síntomas
COMPROMISO GLOMERULAR EN PHS
INMUNOFLUORESCENCIA
MICROSCOPIA ELECTRÓNICA
LABORATORIO
• Leucocitos ligeramente altos, VSG normal
• Plaquetas normales, pueden estar
aumentadas en pacientes con dolor
abdominal o hemorragia GI
• ANA y FR negativos
• IgA aumentada en el 50% de los casos con
enfermedad aguda
HISTOPATOLOGÍA
• Típica: vasculitis leucocitoclastica con o sin
necrosis fibrinoide
• Menos especifica: infiltrado linfohistiocitico
perivascular con extravasación de eritrocitos
• Niños mayores IgA 100% en niños pequeños
es raro: C3 e IgM más común
VASCULITIS LEUCOCITOCLASTICA EN
PHS
TRATAMIENTO
• Corticoides: disminuyen la duración del dolor
abdominal y la artritis
• Dapsona: acorta la duración del rash, inhibe la
interacción IgA – neutrofilos
• Factor XIII: útil en sangrado GI
• Plasmaferesis e inmunoglobulinas: Enfermedad grave
• Inmunosupresores: azatropina, ciclofosfamida,
micofenolato
PRONÓSTICO
• Por lo general es una enfermedad aguda
autolimitada
• El pronóstico se ensombrece con el
compromiso renal
• 33% de los casos presentan recurrencia entre
las 2 semanas y los 18 meses
CLAVE C
Preguntas de Pediatría
72. Preescolar de 4 años presenta desde hace 24
horas petequias en nalgas y extremidades
inferiores, hemorragia gingival y de mucosas. ¿Cuál
es el diagnostico probable?
A. Purpura de Schonlein-Henoch
B. Purpura trombotica trombocitopenica
C. Purpura trombocitopenica idiopatica
D. Sindrome urémico hemolítico
E. Destruccion plaquetaria no inmunitaria
PÚRPURA TROMBOPÉNICA IDIOPÁTICA
(PTI)

• Destrucción plaquetar por


mecanismo autoinmune.
• Suele existir antec. de infección
viral 1-3 semanas antes.
• Ac antiplaquetarios de tipo IgG
frente a glicoproteínas de la mb
plaquetaria con secuestro y
destrucción de las plaquetas por
el sistema mononuclear
fagocítico (bazo).
CLÍNICA

• Comienzo brusco.
• De 2-6 años en primavera e invierno.
• Equimosis y exantema petequial generalizado y asimétrico
• BEG
• No adenopatías ni visceromegalias.
• Puede existir epistaxis, gingivorragia, hematuria, hematemesis
y melenas. Hemorragia Intracraneal en <1% (causa + frec de
mortalidad)
• Síndrome de Evans: PTI + anemia hemolítica autoinmune
EXPLORACIONES COMPLEMENTARIAS
• Hemograma: trombopenia con aumento del VPM. Puede existir
anemia (epistaxis severa, S. Evans), linfocitosis relativa y
eosinofilia.
• Estudio de coagulación y Ac antifosfolípido.
• Grupo sanguíneo, Rh, Coombs directo
• Ig, subpoblaciones linfocitarias, AC antinucleares.
• Serología toxoplasma, CMV, VEB, Parvovirus B19, VHS, VVZ,
VHA, B y C
• BQ
• Medulograma
• Sed orina (hematuria microscópica)
• AC antiplaquetarios.
DIAGNÓSTICO

• Se sospecha:
– Niño con púrpura, BEG, exploración física normal.
– Trombopenia aislada
– Normalidad del resto de las series.
– Estudio de la coagulación normal.
• Se confirma:
– Medulograma con celularidad normal (nº normal o ↑ de
megacariocitos).
• Indicado en todos los niños con clínica que no sea típica,
anomalías en el hemograma y especialmente si se inicia
tratamiento con corticoides
– Los Ac antiplaquetarios + confirman el diagnóstico, pero su
negatividad no lo excluye.
CLAVE C
Preguntas de Pediatría
73.Varón de 13 años que hace 2 semanas presentó
proceso viral. Ingresa a emergencia por presentar
petequias en abdomen. Al examen no se encuentra
viceromegalia ni adenopatías. Examen de laboratorio:
plaquetas 90,000/mm3 ¿cuál será la conducta a seguir?
A. Reposición de plaquetas
B. Administración de glucocortidoides
C. Observación
D. Indicación de inmunoglobulina intravenosa
E. Administración de globulina anti Rh (D)
CLASIFICACIÓN SEGÚN EVOLUCIÓN

PTI AGUDA: Duración < 6m.


• Simple (80-85%): un solo brote, de curso corto y favorable,
incluso sin tratamiento.
• Recidivante (4%): se produce una recaída tras nua
normalización de más de 6 semanas sin tto.

PTI CRÓNICA: Duración > 6m. (10%).


• Más frecuente en niños >10a, formas de inicio insidioso, con
antecedentes viral y con recuentos entre 20,000 y
100,000/mm3.
• Un 30% puede tener remisiones espontáneas dentro del 1º
año.
TRATAMIENTO

• RECOMENDACIONES GENERALES:
– Ingreso si precisa transfusión o si plaquetas <
20.000/mm3.
– Transfusión de plaquetas: sólo en urgencias con riesgo
vital (plaquetas entre 5.000-20.000/mm3 con sangrado
visceral o plaquetas < 5.000/mm3).
– Siempre que la situación clínica lo permita hay que
posponer el tratamiento corticoideo hasta que se realice
medulograma.
– Evitar inyecciones IM., F q alteren la función plaquetaria
(aspirina, AINEs, antihistamínicos,...).
– Escolarización normal si recuento > 20.000/mm3, pero
con restricción de la actividad física intensa y deportes
de contacto.
TRATAMIENTO ESPECÍFICO
• Ɣ-globulina: cuando sea necesario un ↑
rápido de la cifra de plaquetas (sangrado,
ttismo, cirugía).
• Corticoides: tratamiento de elección.
CLAVE C
Preguntas de Pediatría
74. Niño de 7 años, presenta hace 2 días edema de
miembros inferiores. Hoy amanece con edema
bipalpebral, orina de color rojiza. Como antecedente hace
2 semanas fue diagnosticado de faringitis aguda. ¿Cuál es
el diagnóstico más probable?
A. Glomerulonefritis postinfecciosa
B. Nefropatía por Ig A
C. Síndrome Nefrótico
D. Glomerulonefritis membranosa
E. Síndrome Urémico hemolítico
Epidemiología del Síndrome Nefrítico
• Hematuria, hipertensión y edemas de forma
aguda
• Las causas más frecuentes son las
postinfecciosas que producen síndrome
nefrítico agudo
• Dentro de éstas, la más frecuentemente
observada es la GNAPE.
Organismos implicados como factores
etiológicos
• BACTERIAS: VIRUS
• ESTREPTOCOCO,GRUPO A b-hemolitico. VARICELLA.
• ESTREPTOCOCO VIRIDANS. RUBEOLA
• ESTREPTOCOCO PNEUMONIAE. CITOMEGALOVIRUS.
• ESTAFILOCOCO AUREUS. EBSTEIN-BARR VIRUS.
• ESTAFILOCOCO EPIDERMIDIS. PARASITOS
• CORINEBACTERIAS. TOXOPLASMA .
• PROPIONIBACTERIAS TRICHINELLA.
• MICOBACTERIAS ATIPICAS. RICKETTSIA.
• MICOPLASMA
• BRUCELLA
• MENINGOCOCO.
• LEPTOSPIRA.
Antecedente de infección
estreptocócica
• El riesgo de sufrir nefritis postestreptocócica después
de una infección por un estreptococo nefritogénico
es del 15%.
• Las infecciones estreptocócicas de garganta
asociadas a GNAPE pueden manifestarse con un
cuadro clínico variable
• Molestias leves en la garganta, hasta exudado
purulento en asociación con fiebre y adenitis cervical
Antecedente de infección estreptocócica

• La utilidad del cultivo está en definir si la gran


mayoría de faringitis, que de hecho presentan
uno o dos de los hallazgos mencionados,
deben recibir tratamiento.
• El período de latencia entre las infecciones en
la garganta y el inicio de la nefritis suele ser de
2-3 semanas.
Antecedente de infección estreptocócica

• El impétigo estreptocócico se caracteriza por


la formación de vesículas
• Generalmente se rompen con facilidad
dejando costras
• Presencia de linfadenopatías regionales
Antecedente de infección estreptocócica

• Con frecuencia, la piodermitis se instala sobre


lesiones de escabiosis.
• El período de latencia en la glomerulonefritis
postimpétigo es mayor que el de las faringitis
(3-6 semanas).
Diagnóstico
• El estudio de la etiología estreptocócica de la
nefritis debe comenzar con un buen
interrogatorio clínico haciendo énfasis en
antecedentes clínicos de infección
estreptocócica.
• Debe examinarse el paciente en busca de
piodermitis, otitis o faringitis asociadas.
Diagnóstico
• En caso de existir infección activa deben
tomarse cultivos
• En muchos casos no se puede precisar el sitio
de la infección, por ello siempre es necesario
estudiar los anticuerpos sericos
antiestreptococo
CLAVE A
Preguntas de Pediatría
75. Niño de 6 años de edad con antecedente de
faringitis a repetición, acude por presentar cefalea,
PA: 120/90 mmHg, edema palpebral y escaso
volumen urinario. ¿Cuál es el tratamiento más
adecuado?
A. Restricción de líquidos
B. Captopril
C. Furosemida
D. Resinas de intercambio iónico
E. Diálisis peritoneal
Tratamiento
• Debe indicarse la restricción en la ingesta
de agua y en el sodio en todos los
pacientes con SNA.
• Durante las primeras 12-24 horas, cuando
no se conoce la diuresis, es razonable
omitir todas las ingestas orales a fin de
establecer un equilibrio negativo y evaluar
la diuresis
Tratamiento
• No se ha demostrado que el reposo absoluto
mejora la evolución clínica de la enfermedad
• Es recomendable el reposo relativo y los
pacientes lo guardan por no sentirse bien
durante la fase aguda
• El uso de diuréticos de asa produce un
aumento rápido del volumen de orina en el
80% de los pacientes con GNAPE
Tratamiento
• Es raro que se necesite su uso durante más de
48 horas.
• Varios estudios han demostrado que la
furosemida aumenta 4-9 veces la diuresis, con
lo cual se reduce el tiempo de normalización
de la tensión arterial y la desaparición total
del edema (de 7-10 días a 4-5 días).
Tratamiento
• Otros diuréticos como las tiacidas no son
efectivos
• Los inhibidores de la aldosterona como la
espironolactona están contraindicados por el
peligro de hipercalemia
Fármacos antihipertensivos
• Son necesarios en el 50% de los pacientes, y
en raras ocasiones se requiere su uso más de
2-3 días en enfermos con GNAPE.
• Nifedipino 0,25 a 0,5 mg/Kg./dosis VO/SL cada
6-8 hrs. Máximo 10mg/dosis o 3mg/kg/día
Fármacos antihipertensivos
• Los IECA deben evitarse porque producen
hiperpotasemia
• Los bloqueadores beta tampoco deben utilizarse
porque el estado de hipervolemia en que se
encuentra el paciente hace peligroso el desarrollo de
una insuficiencia cardíaca y peligro de
hiperpotasemia.
• El uso de digitálicos está contraindicado porque no
son efectivos en la insuficiencia cardíaca y la
intoxicación es frecuente.
CLAVE C
Preguntas de Pediatría
76. Una niña de 7 años presentó hace 2 meses edemas, hipertensión,
hematuria con 80% de crenocitos, C3 disminuido y con ASO
incrementado asociado a faringoamidalitis supurada 2 semanas
previas al inicio de los síntomas. Acude a control por consultorio
donde observa C3 y ASO en valores normales examen de orina
leucocitos 0-1/campo, GR de 10-15/campo, proteinuria (-) que haría
usted :
A. Solicitaría biopsia renal
B. Iniciaría corticoides por riesgo de glomerulopatia secundaria
C. Esperaría hasta un año el cese de la hematuria
D. Solicitaría una urografía excretoria
E. Solicitaría una Tomografía espiral multicorte de vías urinarias
Indicaciones de biopsia renal en la GNAPE

• Proteinuria persistente en rango nefrótico


• Cuadro clínico estacionario después de 2
semanas de evolución, en especial si existe un
período de anuria prolongado
• Concentraciones normales de complemento
sérico
Indicaciones de biopsia renal en la GNAPE

• Falta de evidencias serológicas o clínicas de


infección estreptocócica
• Complemento sérico bajo después de la sexta
semana
• Cuando persiste o reaparece la proteinuria
después de los 6 meses del episodio agudo
CLAVE C
Preguntas de Pediatría
77. La característica principal del síndrome
nefrótico es:
A. Proteinuria hipoproteinemia
B. Disminución del complemento
C. Formación de complejos inmunológicos en
glomérulos
D. Disminución del colesterol
E. Hematuria
CLAVE A
Preguntas de Pediatría
78. ¿Cuál no es una indicación de biopsia renal
en síndrome nefrótico?
A. Niños menores de 12 meses
B. Niños mayores de 10 años
C. Sospecha de enfermedad sistémica.
D. Dos recaídas en un año.
E. Hipocomplementemia persistente.
Antes de la terapia inicial
• En niños < 12 meses
• En niños > 10 años
• Sospecha de enfermedad sistémica.
• En Insuficiencia Renal, HTA y/o hematuria
constante e importante.
• La presencia de hipocomplementemia persistente.
• Signos analíticos de una tubulopatía proximal
Después de la terapia inicial
• Corticorresistentes
• Corticodependientes
• Recaedores frecuentes
• Antes del tratamiento con agentes
nefrotóxicos como ciclosporina A o
tacrolimus
ESQUEMA DE TERAPIA EN
SÍNDROME NEFRÓTICO CORTICOSENSIBLE

INICIAL SEGUIMIENTO SUSPENSIÓN

60mg/m2/día 40mg/m2/día
2 dosis al día 1 dosis cada 48 horas Disminuir
6 semanas 6 semanas 5 mg/semanal

ESQUEMA DE TERAPIA EN SÍNDROME


NEFRÓTICO CON RECAÍDAS INFRECUENTES
Prednisona 60 mg/m2/día dividido en 2 dosis (dosis máx 80 mg)
hasta la obtención de remisión completa
Seguido de prednisona alterna 40 mg/m2/48 h (dosis máx 60 mg) por
4 semanas
CLAVE D
Preguntas de Pediatría
79. Varón de 12 años, 30 Kg de peso, edema facial y luego
de miembros inferiores desde hace un mes. Proteinuria
en 24 horas 2.4 gramos al día, no hematuria, la presión
arterial es normal. Biopsia renal: microscopia de luz e
inmunofluorescencia normal. ¿Cuál es la nefropatía?
A. proliferativa extracapilar
B. membranoproliferativa
C. proliferativa endocapilar
D. membranosa
E. por cambios minimos
Definición de Síndrome Nefrótico
• Presentación clínica con edema y oliguria
• Proteinuria mayor de 40 mg/m2/h
• Hipoalbuminemia inferior de 2,5 g/dl
• Hipercolesterolemia

International Study of Kidney Disease in Children: Nephrotic Syndrome in Children: Prediction of histopathology from clinical and laboratory characteristics at
time of diagnosis. Kidney Int 1978; 13: 159-165.
Epidemiología
• El 90% de casos de SN está asociado a causas
primarias, siendo el resto de etiología
secundaria
• El 80% de los niños con SN primario es
corticosensible

International Study of Kidney Disease in Children: Nephrotic Syndrome in Children: Prediction of histopathology from clinical and laboratory characteristics at
time of diagnosis. Kidney Int 1978; 13: 159-165.
Correlación clínico patológica del Síndrome
Nefrótico Primario

Síndrome Nefrótico Primario


100%

Cambios Mínimos GEFS MP y otras


80% 10% 10%

Corticosensibles 75% Corticosensibles 2% Corticosensibles 2%


Corticoresistentes 15% Corticoresistentes 8% Corticoresistentes 8%

International Study of Kidney Disease in Children: Nephrotic Syndrome in Children: Prediction of histopathology from clinical and laboratory characteristics at
time of diagnosis. Kidney Int 1978; 13: 159-165.
Remisión completa
• Proteinuria < 4 mg/ m2/h
• Proteinuria / creatinuria < 0,2
• Cinta reactiva (-) o trazas durante 3 días
consecutivos
• Albúmina > 3.5 gr/dL
• Ausencia de edemas

International Study of Kidney Disease in Children: Nephrotic Syndrome in Children: Prediction of histopathology from clinical and laboratory characteristics at
time of diagnosis. Kidney Int 1978; 13: 159-165.
Definiciones
• SN corticosensible: Remisión completa a las 8
semanas de terapia estándar con esteroides
• SN recaedor frecuente: ≥ 2 recaídas en 6 meses
después del episodio inicial o ≥ 4 recaídas en
cualquier período de 12 meses
• SN corticodependiente: ≥ 2 recaídas durante la
terapia esteroidea o dentro de los 14 días posteriores
a la suspensión de ella

International Study of Kidney Disease in Children: Nephrotic Syndrome in Children: Prediction of histopathology from clinical and laboratory characteristics at
time of diagnosis. Kidney Int 1978; 13: 159-165.
SN Corticoresistente
• Falta de remisión tras 8 semanas de
tratamiento
• 4 semanas de prednisona a 60mg/m2/d
seguido de 4 semanas de prednisona en días
alternos a una dosis de 40 mg/m2/día

International Study of Kidney Disease in Children: Nephrotic Syndrome in Children: Prediction of histopathology from clinical and laboratory characteristics at
time of diagnosis. Kidney Int 1978; 13: 159-165.
FLUJOGRAMA DEL PROCESO DE ATENCIÓN DE NIÑOS CON
SINDROME NEFRÓTICO
CLAVE E
Preguntas de Pediatría
80. En el síndrome nefrótico existe una retención renal de
sodio y agua e hipoalbuminemia. ¿Cuál de los siguientes
factores explica que este exceso de líquido no
permanezca en el compartimiento vascular y pase al
intersticial en forma de edema:
A. El aumento generalizado de la permeabilidad de los
capilares
B. El descenso de la presión oncótica del plasma
C. La disminución de la presión osmótica intracapilar
D. El aumento de la presión hidrostática intracapilar
E. B y D
Mecanismo de formación de edemas en SN

Underfill Overfill

Rondon-Berrios H: New insights into the pathophysiology of oedema in nephrotic syndrome. Revista Nefrología 2011; 31(2):148-54.
Argumentos en contra de la hipótesis del
underfill de la formación del edema
• Pacientes y ratas con niveles bajos de • Los volúmenes plasmático y
albúmina sérica no desarrollan sanguíneo son normales o se
edema ni retención de sodio encuentran incrementados en el
• La natriuresis en la fase de síndrome nefrótico.
recuperación del síndrome nefrótico • Expansión del espacio intravascular
comienza cuando la proteinuria con albúmina no aumenta la
desaparece, pero antes de que la natriuresis en pacientes con
albúmina sérica vuelva a la síndrome nefrótico.
normalidad • La activación del SRAA no está
• La disminución absoluta en la presión implicada en la generación del
oncótica plasmática no afecta al edema en el síndrome nefrótico.
volumen sanguíneo en el síndrome • La adrenalectomía bilateral no
nefrótico previene la retención de sodio en el
síndrome nefrótico experimental en
ratas

Rondon-Berrios H: New insights into the pathophysiology of oedema in nephrotic syndrome. Revista Nefrología 2011; 31(2):148-54.
Mecanismo de formación de edemas en SN
Jv = Kf ( [Pc - Pi ] - σ[πc - πi] )

Siddall, E.C. & Radhakrishnan, J. 2012. The pathophysiology of edema formation in the nephrotic syndrome. Kidney Int 82, 635–642.
Mecanismo de formación de edemas en SN

Siddall, E.C. & Radhakrishnan, J. 2012. The pathophysiology of edema formation in the nephrotic syndrome. Kidney Int 82, 635–642.
Mecanismo de formación de edemas en SN
• Aunque la retención de Na tiene un papel preponderante
es improbable que sea el único factor ( Síndrome de
Liddle no causa edemas)
• Es posible que una reducción crítica de la gradiente de la
presión oncótica capilar-intersticial o anomalías en la
permeabilidad vascular sean importantes
• Una hipoproteinemia severa aguda ( Cambios Mínimos,
plasmaféresis) pueden causar edema y depleción del
intravascular

Siddall, E.C. & Radhakrishnan, J. 2012. The pathophysiology of edema formation in the nephrotic syndrome. Kidney Int 82, 635–642.
CLAVE E

You might also like